Anda di halaman 1dari 82

AE06/AC04/AT04

SIGNALS & SYSTEMS

TYPICAL QUESTIONS & ANSWERS


PART I

OBJECTIVE TYPE QUESTIONS


Each Question carries 2 marks. Choose the correct or best alternative in the following: Q.1 The discrete-time signal x (n) = (-1) is periodic with fundamental period (A) 6 (B) 4 (C) 2 (D) 0 Ans: C Period = 2
n

Q.2

The frequency of a continuous time signal x (t) changes on transformation from x (t) to x ( t), > 0 by a factor 1 (A) . (B) .

(C) .
2

(D) Transform

Ans: A x(t) >1 <1

x(t), > 0 compression in t, expansion in f by . expansion in t, compression in f by .

Q.3

A useful property of the unit impulse (t) is that (B) (at) = (t) . (A) (at) = a (t) . 1 (C) (at) = (t) . (D) (at ) = [(t )]a . a Ans: C Time-scaling property of (t): (at) = 1 (t), a > 0 a
The continuous time version of the unit impulse (t) is defined by the pair of relations

Q.4

AE06/AC04/AT04
1 (A) (t) = 0

SIGNALS & SYSTEMS

t=0 (B) (t) = 1, t = 0 and (t) dt = 1 . t 0 . - 1, t 0 (C) (t) = 0, t 0 and (t) dt = 1 . (D) (t ) = . - 0, t < 0
Ans: C (t) = 0, t 0 (t) 0 at origin
+

(t) dt = 1 Total area under the curve is unity.


-

[(t) is also called Dirac-delta function]

Q.5

Two sequences x1 (n) and x2 (n) are related by x2 (n) = x1 (- n). In the z- domain, their ROCs are (A) the same. (B) reciprocal of each other. (C) negative of each other. (D) complements of each other. z Ans: B x1(n) X1(z), RoC Rx z Reciprocals x2(n) = x1(-n) X1(1/z), RoC 1/ Rx

Q.6 The Fourier transform of the exponential signal e j 0 t is (A) a constant. (B) a rectangular gate. (C) an impulse. (D) a series of impulses. Ans: C Since the signal contains only a high frequency o its FT must be an impulse at = o Q.7 If the Laplace transform of f (t ) is (A) cannot be determined. (C) is unity.
L
s2 + 2

(s

, then the value of Lim f (t ) t + 2 (B) is zero. (D) is infinity.

Ans: B f(t)

Lim f(t) = Lim t s 0 =


s

s F(s) s s2 + 2

[Final value theorem]

Lim
0

=0

Q.8 The unit impulse response of a linear time invariant system is the unit step function u (t ) . For t > 0, the response of the system to an excitation

e at u (t ), a > 0, will be
(A) ae at . 2 (B)
1 e at . a

AE06/AC04/AT04 (C) a (1 e at ) . Ans: B h(t) = u(t); x(t) = e-at u(t), a > 0


System response y(t) = L1 s . s + a 1 1 1 1 1 = L1 a s s + a

SIGNALS & SYSTEMS (D) 1 e at .

= 1 (1 - e-at) a

Q.9 The z-transform of the function (A) z > 1 (C) z < 1


0

(n k ) has the following region of convergence


k =

(B) z = 1 (D) 0 < z < 1


x(n) =
0

Ans:

C
x(z) =

(n-k)
k = -

z-k = ..+ z3 + z2 + z + 1 (Sum of infinite geometric series)

k = -

= 1 , 1z

z <1

Q.10 The auto-correlation function of a rectangular pulse of duration T is (A) a rectangular pulse of duration T. (B) a rectangular pulse of duration 2T. (C) a triangular pulse of duration T. (D) a triangular pulse of duration 2T. Ans: D
T/2

RXX () = 1 x() x(t + ) d T -T/2

triangular function of duration 2T.

Q.11 The Fourier transform (FT) of a function x (t) is X (f). The FT of dx (t ) / dt will be (A) dX(f ) / df . (B) j2f X(f ) . (C) jf X (f ) . (D) X (f ) / ( jf ) . Ans: B (t) = 1 X(f) ejt d 2 -
d x = 1 j X(f) ejt d dt 2 - d x j 2 f X(f) dt

Q.12 The FT of a rectangular pulse existing between t = T / 2 to t = T / 2 is a (A) sinc squared function. (B) sinc function. (C) sine squared function. (D) sine function. 3

AE06/AC04/AT04
-T t T 2 2 otherwise
e-jt dt = e-jt -T/2 j
+T/2 +T/2

SIGNALS & SYSTEMS

Ans: B x(t) = 1,
0,
+

X(j) = x(t) e-jt dt =


-

-T/2

= - 1 (e-jT/2 - ejT/2) = 2 j = 2 sin T = sin(T/2) .T 2 T/2

ejT/2 - e-jT/2 2j

Hence X(j) is expressed in terms of a sinc function.

Q.13

An analog signal has the spectrum shown in Fig. The minimum sampling rate needed to completely represent this signal is (A) 3 KHz . (B) 2 KHz . (C) 1 KHz . (D) 0.5 KHz .

Ans: C

For a band pass signal, the minimum sampling rate is twice the bandwidth, which is 0.5 kHz here.

Q.14 A given system is characterized by the differential equation:


d 2 y(t ) dy(t ) 2 y(t ) = x (t ) . The system is: dt dt 2 (A) linear and unstable. (B) linear and stable. (C) nonlinear and unstable. (D) nonlinear and stable.

Ans:A

d2y(t) dy(t) 2y(t) = x(t), x(t) x(t) y(t) h(t) dt2 dt system The system is linear . Taking LT with zero initial conditions, we get s2Y(s) sY(s) 2Y(s) = X(s) 1 = 1 or, H(s) = Y(s) = 2 X(s) s s 2 (s 2)(s + 1) Because of the pole at s = +2, the system is unstable.

Q.15 The system characterized by the equation y(t ) = ax (t ) + b is (A) linear for any value of b. (B) linear if b > 0. (C) linear if b < 0. (D) non-linear. 4

AE06/AC04/AT04

SIGNALS & SYSTEMS

Ans: D The system is non-linear because x(t) = 0 does not lead to y (t) = 0, which is a violation of the principle of homogeneity. Q.16 Inverse Fourier transform of u () is 1 1 (A) (t ) + . 2 t 1 (C) 2(t ) + . t
FT

(B)

1 (t ) . 2

(D) (t ) + sgn (t ) .

Ans: A x(t) = u(t)

X(j) = () + J

Duality property: X(jt) u() 1 (t) + 1 2 t

2 x(-)

Q.17 The impulse response of a system is h (n ) = a n u (n ) . The condition for the system to be BIBO stable is (A) a is real and positive. (B) a is real and negative. (C) a > 1 . (D) a < 1 .
+ +

Ans: D Sum S =

h(n) =
n = - +

an u(n)

n = -

a
n=0

u(n) = 1 for n 0 )

1 1- a

if a < 1.

Q.18 If R1 is the region of convergence of x (n) and R 2 is the region of convergence of y(n), then the region of convergence of x (n) convoluted y (n) is (A) R1+ R 2 . (B) R1R 2 . (C) R 1 R 2 . (D) R1R 2 . z Ans:C x(n) X(z), RoC R1 z y(n) Y(z), RoC R2 z x(n) * y(n) X(z).Y(z), RoC at least R1 R2 Q.19
The continuous time system described by y(t ) = x t 2 is (A) causal, linear and time varying. (B) causal, non-linear and time varying. (C) non causal, non-linear and time-invariant. (D) non causal, linear and time-invariant.

( )

AE06/AC04/AT04 Ans: D y(t) = x(t2)


y(t) depends on x(t2) i.e., future values of input if t > 1. System is anticipative or non-causal x1(t) y1(t) = x1(t2) x2(t) y2(t) = x2(t2)

SIGNALS & SYSTEMS

x1(t) + x2(t) y(t) = x1(t2) + x2(t2) = y1(t) + y2(t) System is Linear System is time varying. Check with x(t) = u(t) u(t-z) y(t) and x1(t) = x(t 1) y1(t) and find that y1(t) y (t 1).

Q.20

If G(f) represents the Fourier Transform of a signal g (t) which is real and odd symmetric in time, then G (f) is (A) complex. (B) imaginary. (C) real. (D) real and non-negative. FT

Ans: B g(t)

G(f)

g(t) real, odd symmetric in time G*(j) = - G(j); G(j) purely imaginary.

Q.21

For a random variable x having the PDF shown in the Fig., the mean and the variance are, respectively, (A) 1 and 2 . 2 3 (B) 1 and 4 . 3 2 (C) 1 and . 3 (D) 2 and 4 . 3
+

Ans:B Mean = x(t) = x fx(t) (x) dx


- 3

= x 1 dx = 1 x2 3 = 9 1 1 = 1 -1 4 4 2 -1 2 2 4
+

Variance = (x - x)2 fx (x) dx


- 3

(x - 1)2 1 d(x-1) -1 4

AE06/AC04/AT04
= 1 (x - 1)3 3 = 1 [8 + 8] = 4 -1 12 3 4 3

SIGNALS & SYSTEMS

Q.22

If white noise is input to an RC integrator the ACF at the output is proportional to . (A) exp (B) exp . RC RC (D) exp(- RC ) . (C) exp( RC ) .

Ans: A
RN() = N0 exp - 4RC RC

Q.23

x (n ) = a , a < 1 is (A) an energy signal. (B) a power signal. (C) neither an energy nor a power signal. (D) an energy as well as a power signal.
n

Ans: A + Energy = x2(n) = a2


n=- n=-

= (a2)
n=-

= 1+ 2 a2
n=1

= finite since This is an energy signal.

a <1

Q.24

The spectrum of x (n) extends from o to + o , while that of h(n) extends from 2 o to + 2 o . The spectrum of y(n ) = from

h(k ) x(n k ) extends


k =

(A) 4 o to + 4 o . (C) 2 o to + 2 o .
. Ans: D Spectrum depends on H( ej)

(B) 3 o to + 3 o . (D) o to + o
X( ej) Smaller of the two ranges.

Q.25

The signals x1 (t ) and x 2 (t ) are both bandlimited to ( 1 , + 1 ) and ( 2 , + 2 ) respectively. The Nyquist sampling rate for the signal x1 (t ) x 2 (t ) will be (A) 21 if 1 > 2 . (B) 2 2 if 1 < 2 . (C) 2 (1 + 2 ) . (D) (1 + 2 ) . 2

Ans: C Q.26

Nyquist sampling rate = 2(Bandwidth) = 2(1 (-2)) = 2(1 + 2)

If a periodic function f(t) of period T satisfies f (t ) = f t + T series expansion,

), then in its Fourier

AE06/AC04/AT04 (A) the constant term will be zero. (B) there will be no cosine terms. (C) there will be no sine terms. (D) there will be no even harmonics. Ans:
T T/2 T T/2 0

SIGNALS & SYSTEMS

T/2 0

1 f(t) dt = 1 f(t) dt + f(t) dt = 1 T 0 T 0 T/2 T

f(t) dt + f( + T/2)d = 0

Q.27 A band pass signal extends from 1 KHz to 2 KHz. The minimum sampling frequency needed to retain all information in the sampled signal is (A) 1 KHz. (B) 2 KHz. (D) 4 KHz. (C) 3 KHz. Ans: B Minimum sampling frequency = 2(Bandwidth) = 2(1) = 2 kHz Q.28
The region of convergence of the z-transform of the signal 2 n u (n ) 3n u ( n 1) (A) is z > 1 . (B) is z < 1 .

(C) is 2 < z < 3 . Ans:


2nu(n) 3n u(-n-1) 1 , z >2 1 2 z -1 1 , z <3 1 3z -1

(D) does not exist.

ROC is 2 < z < 3.

Q.29

The number of possible regions of convergence of the function

(e 2 2)z is (z e 2 )(z 2)

(A) 1. (C) 3.

(B) 2. (D) 4.

Ans: C Possible ROCs are z > e-2 , z < 2 and e-2 < z < 2 Q.30
The Laplace transform of u(t) is A(s) and the Fourier transform of u(t) is B( j) . Then 1 1 (A) B( j) = A (s ) s = j . (B) A(s ) = but B( j) . s j 1 1 1 1 (C) A(s ) but B( j) = . (D) A(s ) but B( j) . s j s j L Ans: B u(t) A(s) = 1 s 8

AE06/AC04/AT04
F.T u(t) B(j) = 1 + () j

SIGNALS & SYSTEMS

A(s) = 1 but B(j) 1 s j

Q.31

Given a unit step function u(t), its time-derivative is: (A) a unit impulse. (B) another step function. (C) a unit ramp function. (D) a sine function.

Ans: A Q.32 The impulse response of a system described by the differential equation d2y + y( t ) = x ( t ) will be dt 2 (A) a constant. (B) an impulse function.. (C) a sinusoid. (D) an exponentially decaying function. Ans: C Q.33
The function

sin(u ) is denoted by: (u ) (A) sin c(u). (C) signum.

(B) sin c(u). (D) none of these.

Ans: C Q.34
The frequency response of a system with h(n) = (n) - (n-1) is given by (A) () - ( - 1). (B) 1 - ej. (C) u() u( -1). (D) 1 e-j.

Ans: D Q.35
The order of a linear constant-coefficient differential equation representing a system refers to the number of (A) active devices. (B) elements including sources. (C) passive devices. (D) none of those.

Ans: D Q.36
z-transform converts convolution of time-signals to (A) addition. (B) subtraction. (C) multiplication. (D) division.

Ans: C Q.37
Region of convergence of a causal LTI system (A) is the entire s-plane. (B) is the right-half of s-plane. (C) is the left-half of s-plane. (D) does not exist.

AE06/AC04/AT04 Ans: B Q.38

SIGNALS & SYSTEMS

The DFT of a signal x(n) of length N is X(k). When X(k) is given and x(n) is computed from it, the length of x(n) (A) is increased to infinity (B) remains N (C) becomes 2N 1 (D) becomes N2

Ans: A Q.39
The Fourier transform of u(t) is 1 (A) . j2f 1 (C) . 1 + j2f

(B) j2f. (D) none of these.

Ans: D Q.40
For the probability density function of a random variable X given by

f x ( x ) = 5e Kx u ( x ) , where u(x) is the unit step function, the value of K is 1 1 (A) (B) 5 25 (C) 25 (D) 5
Ans: D Q.41
The system having input x(n) related to output y(n) as y(n) = log10 x (n ) is: (A) nonlinear, causal, stable. (B) linear, noncausal, stable. (C) nonlinear, causal, not stable. (D) linear, noncausal, not stable.

Ans: A Q.42
To obtain x(4 2n) from the given signal x(n), the following precedence (or priority) rule is used for operations on the independent variable n: (A) Time scaling Time shifting Reflection. (B) Reflection Time scaling Time shifting. (C) Time scaling Reflection Time shifting. (D) Time shifting Time scaling Reflection.

Ans: D Q.43
The unit step-response of a system with impulse response h(n) = (n) (n 1) is: (A) (n 1). (B) (n). (C) u(n 1). (D) u(n).

Ans: B

10

AE06/AC04/AT04 Q.44

SIGNALS & SYSTEMS

If () is the phase-response of a communication channel and c is the channel d() frequency, then = c represents: d (A) Phase delay (B) Carrier delay (C) Group delay (D) None of these.

Ans: C Q.45
Zero-order hold used in practical reconstruction of continuous-time signals is mathematically represented as a weighted-sum of rectangular pulses shifted by: (A) Any multiples of the sampling interval. (B) Integer multiples of the sampling interval. (C) One sampling interval. (D) 1 second intervals.

Ans: B Q.46
dx ( t ) If x (t ) X (s), then is given by: dt

(A)

dX(s) . ds

(B)

X (s) x 1 (0) . s s

(C) sX (s) x (0 ) . Ans: C Q.47

(D) sX(s) sX(0).

The region of convergence of the z-transform of the signal x(n) ={2, 1, 1, 2} is n=0 (A) all z, except z = 0 and z = (B) all z, except z = 0. (C) all z, except z = . (D) all z.

Ans: A Q.48
When two honest coins are simultaneously tossed, the probability of two heads on any given trial is: 3 (A) 1 (B) 4 1 1 (C) (D) 2 4

Ans: D Q.49
Let u[n] be a 1, (A) x[n] = 0, 1, (C) x[n] = 0, unit step sequence. The sequence u[ N n] can n<N 1, (B) x[n] = otherwise 0, n>N 1, (D) x[n] = otherwise 0, be described as nN otherwise nN otherwise

11

AE06/AC04/AT04
1, n N Ans (B) x[n] = 0, otherwise Here the function u(-n) is delayed by N units.

SIGNALS & SYSTEMS

Q.50

A continuous-time periodic signal x(t ) , having a period T, is convolved with itself. The resulting signal is (A) not periodic (B) periodic having a period T (C) periodic having a period 2T (D) periodic having a period T/2

Ans (B) periodic having a period T Convolution of a periodic signal (period T) with itself will give the same period T. Q.51
If the Fourier series coefficients of a signal are periodic then the signal must be

(A) continuous-time, periodic (C) continuous-time, non-periodic

(B) discrete-time, periodic (D) discrete-time, non-periodic

Ans B) discrete-time, periodic This is the property of the discrete-time periodic signal. Q.52
The Fourier transform of a signal x(t ) = e 2t u (t ) is given by 1 2 (A) . (B) 2 j 1 j 1 2 (C) (D) j2 j2 1 Ans (A) 2 j 1 1 . Therefore, FT of u(-t) = . If a function x(t) is multiplied FT u(t) = j j by e 2t , then its FT will be F ( j ) j j 2 . Hence the answer. For the function H ( j ) =

1 2 + 2 j + ( j )
2

Q.53

, maximum value of group delay is

(A) 1 (C) 2

(B) 1/2 (D) 3

Ans None of the given answers is correct. Q.54


A continuous-time signal x(t ) is sampled using an impulse train. If X ( j ) is the Fourier transform of x(t ) , the spectrum of the sampled signal can be expressed as

(A) (C)

X ( j + k ) ( )
s

(B) (D)

X ( jk ) * ( + k )
s k =

X ( j ) * ( + k )
s k =

k =

X ( j ) ( + k )
s

k =

12

AE06/AC04/AT04 Ans (A)

SIGNALS & SYSTEMS

X ( j + k ) ( )
s k =

Since the spectrum consists of various harmonics k = to and discretely spread at an interval of fundamental frequency fs. Hence the answer.

Q.55

The region of convergence of a causal finite duration discrete-time signal is

(A) (B) (C) (D)

the entire z-plane except z = 0 the entire z-plane except z = the entire z-plane a strip in z-plane enclosing j -axis

Ans (A) The entire z-plane except z = 0


X ( z) =

x[n]z
n = n1

n2

. This sum should converge provided each term in the sum is

finite. However, if there is a non-zero causal component for n2>0, then X(z) will have a term involving z-1 and thus ROC cannot include z = 0.

Q.56

Let H e j be the frequency response of a discrete-time LTI system, and H I (e j ) be the frequency response of its inverse. Then,

( )

(A) H (e j )H I (e j ) = 1 (C) H (e j ) * H I (e j ) = 1

(B) H (e j )H I (e j ) = ( ) (D) H e j * H I e j = ( )

( )

( )

Ans (A) H e j H I e j = 1 Since H (e j ) and H I (e j ) are the inverse of each other, their product should equal 1. Q.57
The transfer function of a stable system is H ( z ) = Its impulse response will be n n (A) (0.5) u[n] + (2 ) u[n] 1 1 . + 1 1 0. 5 z 1 2 z 1
n n

( ) ( )

(B) (0.5) u[ n 1] + (2 ) u[n] (D) (0.5) u[ n 1] (2 ) u[n 1]


n n

(C)

(0.5)n u[n] (2)n u[n 1]


n n

Ans (C) (0.5) u[n] (2 ) u[ n 1] (A) and (C) are the possible IFTs of the given system function. However, the system is stable; therefore (C) is the only correct answer. Q.58
The probability cumulative distribution function must be monotone and (A) increasing (B) decreasing (C) non-increasing (D) non-decreasing

Ans (D) non-decreasing The probability cumulative distribution function increases to 1 monotonically and there after remains constant. Q.59
The average power of the following signal is

13

AE06/AC04/AT04

SIGNALS & SYSTEMS

A2 2 (C) AT12

(A)

(B) A 2 (D) A 2T1

Ans: (D)
W=

T1 / 2

T1 / 2

x(t ) 2 dt = A 2 T1

Q.60

Convolution is used to find: (A) The impulse response of an LTI System (B) Frequency response of a System (C) The time response of a LTI system (D) The phase response of a LTI system Ans: (C)
k =

Time response

y ( n ) = x ( n ) * h( n ) =

x(k )h(n k )

k =

Q.61

The Fourier Transform of a rectangular pulse is (A) Another rectangular pulse (B) Triangular pulse (C) Sinc function (D) Impulse. Ans: (C) This can be seen by putting the value of pulse function in the definition of Fourier transform.

Q.62

The property of Fourier Transform which states that the compression in time domain is equivalent to expansion in the frequency domain is (A) Duality. (B) Scaling. (C) Time Scaling. (D) Frequency Shifting. Ans: (B) Substituting the square pulse function f(t) in
F ( j ) =

f (t )e jt dt

gives the sinc function.

Q.63

What is the Nyquist Frequency for the signal

x(t) =3 cos 50t +10 sin 300t cos100t ?

(A) 50 Hz (C) 200 Hz

(B) 100 Hz (D) 300 Hz 14

AE06/AC04/AT04

SIGNALS & SYSTEMS

Ans: (D) Here the highest frequency present in the signal is m = 300 or f m = 150 Hz. Therefore the Nyquist frequency f s = 2 f m = 300 Hz.

Q.64

The step response of a LTI system when the impulse response h(n) is unit step u(n) is (A) n+1 (B) n (C) n-1 (D) n 2 Ans: (A)
y ( n ) = x ( n ) * h ( n) = u ( n) * u ( n ) =

u ( k )u ( n k ) =

u (k )u(n k )
k =0

k =

y (0) = 1, y (1) = 2, y (2) = 3, ...., y (n) = (n + 1) y (n) = (n + 1) .

Q.65

The Laplace transform of u(t) is

(A) (C)

1 s 1 s2

(B) s2 (D) s

Ans: (A) Substituting f (t ) = u (t ) in the relation F ( s ) = f (t )e st dt gives the answer.


o

Q.66

The function which has its Fourier transform, Laplace transform, and Z transform unity is (A) Gausian (B) impulse (C) Sinc (D) pulse Ans: (B) Substituting f(t) = (t ) in the definitions of Fourier, Laplace and Z-transform, we get the transforms in each case as 1.

Q.67

The Z transform of (n m ) is (A) z n

(B) z m (D)
1 zm

(C)

1 z n

Ans: (B) The Z-transform of a delayed function f(n-m) is z-m times the Z-transform of the function f(n).

Q.68

If the joint probability pdf of f (x, y ) = , 0 x, y 2, P(x + y 1) is 1 8 1 (C) 4

1 4

(A)

1 16 1 (D) 2

(B)

15

AE06/AC04/AT04
Ans: (A)
P( x + y ) =

SIGNALS & SYSTEMS

1 1 y

0 0

1 1 dxdy = 4 4

x0
0

1 y

dy =

1 4

(1 y )dy = 8 .
0

Q.69

The period of the signal x(t ) = 10 sin 12t + 4 cos 18t (A) (B) 4 1 (C) (D) 9 Ans: (D)

is 1 6 1 3

There are two waveforms of frequencies 6 and 9, respectively. Hence the combined frequency is the highest common factor between 6 and 9,i.e., 3. Hence period is 1/3.

Q.70

The autocorrelation of a rectangular pulse is (A) another rectangle pulse (C) Triangular pulse Ans: (C)

(B) Square pulse (D) Sinc pulse

Autocorrelation involves the integration of a constant which gives a ramp function. Hence the triangular pulse.

Q.71

If the Fourier series coefficients of a signal are periodic then the signal must be (A) continuous-time, periodic (B) discrete-time, periodic (C) continuous-time, non periodic (D) discrete-time, non perodic Ans: (B) It is the property of the discrete-time periodic signal.

Q.72

The area under the curve

(t ) dt is

(A) (C) 0
Ans: (B)

(B) unity (D) undefined

(t ) = 1

By definition of delta function,

Q.73

A transmission is said to be _____________ if the response of the system is exact replica of the input signal.

(A) LTI (C) Distortionless


Ans: (C) Since y (n) = x(n) .

(B) Distorted (D) Causal

16

AE06/AC04/AT04 Q.74
Laplace Transform of tn is always equal to n n! (A) n (B) n s s n! (C) n +1 (D) All s Ans: (C)
tn =

SIGNALS & SYSTEMS

t n e st dt =

n! s n +1

Q.75

For a stable system

(A) (C)

z <1 z >1

(B) (D)

z =1 z 1

Ans: (A) For the system to be stable, the ROC should include the unit circle.

Q.76

The region of convergence of a causal finite duration discrete time signal is (A) The entire z plane except z = 0 (B) The entire z plane except z = (C) The entire z plane (D) A strip in z-plane Ans: (A) The ROC of the causal finite duration will have negative power of z. The ROC is the entire z-plane except z = 0.

Q.77

The CDF for a certain random variable is given as

< x0 0, 2 F X ( x ) = kx , 0 < x 10 100k , 10 < x < The value of k is (A) 100 (B) 50 (C) 1/50 (D) 1/100 Ans: (D) From the given F(x), we get
dF ( x ) = 0 + 2kx + 0 = 2kx dx 2kxdx = 1
0

10

or 100k = 1 k = 1 / 100

Q.78

The group delay function () is related to phase function () as d d (A) ( ) = ( ) (B) ( ) = ( ) d d 2

17

AE06/AC04/AT04 (C) ( ) =
d2 ( ) d 2

SIGNALS & SYSTEMS (D) ( ) =


d2 ( ) d

Ans: (A): By definition.

Q.79

Two sequences x1 (n ) and x 2 (n ) are related by Z-domain, their ROCs are (A) same (C) negative of each other Ans: (B)

x 2 (n ) = x1( n ) . In the
(B) reciprocal of each other (D) complement of each other

ROC of Z [x2 (n)] is outside the circle of radius r2 while ROC of Z [x1 (n)] is inside the circle of radius r1 such that r2 = 1/ r1.

Q.80

The autocorrelation of a sinusoid is (A) Sinc pulse (C) Rectangular pulse Ans: (B)
XX (t ) = x( ) x( t )d
= =

(B) another sinusoid (D) Triangular pulse

A sin A sin ( t )d

A2 [(cos t cos 2 . cos t sin 2 . sin t ]d 2 A2 = K [(cos t cos 2 . cos t sin 2 . sin t ]d 2 2 A = K [cos t ]d = K ' cos t 2 .

Thus the autocorrelation is a sinusoid.

Q.81

Which of the following is true for the system represented by y (n ) = x( n )

(A) Linear (C) Causal


Ans.: (A)

(B) Time invariant (D) Non Linear

The given function is of the form y = mx . Hence linear.

Q.82

The Fourier transform of impulse function is (A) () (B) 2 (C) 1 (D) sinc f Ans: (C) FT of
(t ) = (t )e jt dt = 1

Q.83

Convolution is used to find (A) amount of similarity between the signals 18

AE06/AC04/AT04 (B) response of the system (C) multiplication of the signals (D) Fourier transform
Ans: (B)

SIGNALS & SYSTEMS

Convolution of the input signal x(n) and the impulse response h(n) is given by
y ( n ) = x ( n) * h( n ) =

x ( k ) h(n k )
k =

, where y (n) is the response of the system.

Q.84

The final value of x(t ) = 2 + e 3t u (t ) is (A) 2 (B) 3

(C) e 3t
Ans: (A)

(D) 0

3t Final value = Lt t x(t ) = Lt t 2 + e u (t ) = 2 .

Q.85

Discrete time system is stable if the poles are (A) within unit circle (C) on the unit circle Ans: (A) The ROC should include the unit circle.

(B) outside unit circle (D) None

Q.86

The z transform of u ( n 1) is 1 (A) 1 z 1 (C) 1 z 1 Ans: (C)

(B) (D)

z 1 z z

1 z 1
z 1 = z 1 1 z 1

z[ u (n 1)] = [u (n 1)]z n = [ z + z 2 + z 3 + ...] =


n = 1

Q.87

The area under Gaussian pulse

t2

dt is

(A) Unity (C) Pulse


Ans: (A)

(B) Infinity (D) Zero

Q.88

2 e t dt =

e x 2

dx = 2

x e x dx =1.

The spectral density of white noise is (A) Exponential (C) Poisson

(B) Uniform (D) Gaussian

19

AE06/AC04/AT04
Ans: (B)

SIGNALS & SYSTEMS

The distribution of White noise is homogeneous over all frequencies. Power spectrum is the Fourier transform of the autocorrelation function. Therefore, power spectral density of white noise is uniform.

20

AE06/AC04/AT04 PART II

SIGNALS & SYSTEMS

NUMERICALS & DERIVATIONS


Q.1. Determine whether the system having input x (n) and output y (n) and described by n relationship : y (n) = x (k + 2) is (i) memoryless, (ii) stable, (iii)causal (iv) k =- linear and (v) time invariant. (5) Ans: y(n) =
x(k + 2)
k = -

(i) Not memoryless - as y(n) depends on past values of input from x(-) to x(n-1) (assuming)n > 0) (ii) Unstable- since if x (n) M, then y(n) goes to for any n. (iii) Non-causal - as y(n) depends on x(n+1) as well as x(n+2). (iv) Linear the principle of superposition applies (due to operation) (v) Time invariant - a time-shift in input results in corresponding time-shift in output.

Q.2.

Determine whether the signal x (t) described by x (t) = exp [- at] u (t), a > 0 is a power signal or energy signal or neither.

(5)

Ans: -at x(t) = e u(t), a > 0


x(t) is a non-periodic signal.
+

- at - at

Energy E = x2(t) dt = e 2 dt = e 2 - 0 -2a The energy is finite and deterministic. x(t) is an energy signal.

= 1 (finite, positive) 2a
0

Q.3. Determine the even and odd parts of the signal x (t) given by t>0 A e - t x (t) = t<0 0
x(t) = Ae 0
-t

t>0 t<0

(5)

Ans: Assumption : > 0, A > 0, - < t <


Even part xe(t) = Odd part xo(t) = x(t) + x(-t) 2 x(t) - x(-t) 2

21

AE06/AC04/AT04
x(t) A t 0 x(-t) Ae
+ t

SIGNALS & SYSTEMS

Ae

-t

A t 0 xe(t) A/2 0 xo(t) A/2 t 0 -A/2 t

Q.4.

Use one sided Laplace transform to determine the output y (t) of a system described by d2y dy dy + 3 + 2y (t) = 0 where y (0 ) = 3 and =1 (7) 2 dt dt t =0 dt Ans: d2y + 3 dy + 2 y(t) = 0, y(0-) = 3, dy =1 2 dt dt dt t = 0 s2 Y(s) s y(0) dy + 3 [s Y(s) y(0)] + 2 Y(s) = 0 dt t = 0 (s2 + 3s + 2) Y(s) = sy(0) + dy + 3 y(0) dt t = 0 (s2 + 3s + 2) Y(s) = 3s + 1 + 9 = 3s + 10 Y(s) = 3s + 10 = 3s + 10 s + 3s + 2 (s + 1)(s + 2)
2

= A + B s+1 s+2

22

AE06/AC04/AT04

SIGNALS & SYSTEMS

A = 3s + 10 = 7; s + 2 s = -1 Y(s) = 7 - 4 s+1 s+2

B = 3s + 10 = -4 s + 1 s = -2

y(t) = L-1 [Y(s)] = 7e-t 4e-2t = e-t( 7 - 4e-t) The output of the system is y(t) = e-t( 7 - 4e-t) u(t)

Q. 5. Obtain two different realizations of the system given by y (n) - (a+b) y (n 1) + aby (n 2) = x (n).Also obtain its transfer function. Ans:
y(n) (a + b) y(n-1) + ab y(n-2) = x(n) Y(z) (a+b) z-1 Y(z) + ab z-2 Y(z) = X(z) Transfer function H(z) = Y(z) = 1 X(z) 1 (a+b) z-1 + ab z-2 y(n) = x(n) + (a + b) y(n-1) - ab y(n-2) Direct Form I/II realization Alternative Realisation

(7)

Q. 6.

An LTI system has an impulse response h (t) = exp [ -at] u (t); when it is excited by an input signal x (t), its output is y (t) = [exp (- bt) -exp (- ct)] u (t) Determine its input x (t). (7)

Ans: -at h(t) = e u(t) for input x(t)


Output y(t) = (e L h(t) H(s), y(t)
-bt

- e ) u(t) L Y(s), x(t) L X(s)

-ct

23

AE06/AC04/AT04
H(s) =

SIGNALS & SYSTEMS


1 ; Y(s) = 1 - 1 = s+csb = c-b s+a s+b s+c (s + b)(s + c) (s + b)(s + c)

As H(s) = Y(s) , X(s) = Y(s) X(s) H(s) X(s) = (c - b)(s + a) = (s + b)(s + c) A + B s+b s+c

A = (c - b)(s + a) = (c b)(-b + a) = a - b (s + c) s = -b (-b + c) B= (c - b)(s + a) = (c b)(-c + a) = c - a (s + b) s= -c (-c + b) c-a s+c


-ct -bt -ct

X(s) = a b + s+b x(t) = (a b) e


-bt

+ (c a) e

The input x(t) = [(a b) e

+ (c a) e ] u(t)

Q.7. Write an expression for the waveform f (t ) shown in Fig. using only unit step function and powers of t. (3) Ans:

f(t) = E [ t u(t) 2(t T) u(t T) + 2(t 3T) u(t 3T) (t 4T) u(t 4T)] T

Q.8. For f(t) of Q7, find and sketch f (t ) (prime denotes differentiation with respect to t). Ans:
f(t) = E [ t u(t) 2(t T) u(t T) + 2(t 3T) u(t 3T) (t 4T) u(t 4T)] T

(3)

24

AE06/AC04/AT04
f (t) E/T t 0 -E/T f (t) = E [u(t) 2 u(t - T) + 2 u(t 3T) u(t 4T)] T T 2T 3T 4T

SIGNALS & SYSTEMS

Q.9.

Define a unit impulse function (t ) .

(2)

Ans: Unit impulse function (t) is defined as:


(t) = 0, t 0
+

(t) dt = 1
-

It can be viewed as the limit of a rectangular pulse of duration a and height 1/a when a 0, as shown below.

Q.10. Sketch the function g (t ) =

3
3

(t )2 [u (t ) u (t )] and show that

(6)

g (t ) (t ) as 0 .
Ans:
g(t) 3/ 0

As

0, duration g(t) dt = 1
0

0, amplitude

Q.11.

t Show that if the FT of x (t) is X ( j) , then the FT of x is a X ( ja) . a Ans: FT x(t) X(j)

(6)

25

AE06/AC04/AT04
FT Let x t a
+

SIGNALS & SYSTEMS


X1(j) , then t e-jt dt a
-ja

X1(j) = x
- +

Let t = a

dt = a d

x() e
- +

a d if a> 0 a d if a < 0
+
-

- x() e
-

-ja

Hence X1(j) = a

x() e ja d = a x (ja)
-

Q.12.

Solve, by using Laplace transforms, the following set of simultaneous differential (14) equations for x (t).

Ans: 2 x (t ) + 4 x (t ) + y (t ) + 7 y(t ) = 5u (t ) x (t ) + x (t ) + y (t ) + 3y(t ) = 5(t ) The initial conditions are : x (0 ) = y(0 ) = 0 . 2 x(t) + 4 x(t) + y(t) + 7 y(t) = 5 u(t) x(t) + x(t) + y(t) + 3 y(t) = 5 (t) L L L L s X(s), (t) 1, u(t) x(t) X(s), x(t)
(Given zero initial conditions) 2 sX(s) + 4 X(s) + sY(s) + 7 Y(s) = 5 s sX(s) + X(s) + sY(s) + 3 Y(s) = 5 (2s + 4) X(s) + (s+7) Y(s) = 5 s (s + 1) X(s) + (s+3) Y(s) = 5 X(s) = 5 s+7 S 3 5 s+3 2s+4 s+7 s+1 s+3 Or, X(s) = 5s + 35 5 15/s 2s2 + 6s + 4s + 12 - s2 8s 7 = - 5s2 + 30s 15 = -5 2 s(s + 2s + 5) s s2 + 6s 3 = s2 + 2s + 5

1 s

A + Bs+ C s s2+ 2s +5

Then A (s2+ 2s +5) + B s2 +Cs = -5(s2 + 6s 3)

A +B = -5 2A + C = -30

26

AE06/AC04/AT04
5A =15 Thus A = 3, B = -8, C = -36 and we can write X(s) = 3 8 s +1 14 2 2 2 (s + 1)2 + 22 s (s + 1) + 2 -t -t x(t) = (3 8 e cos 2t 14 e sin 2t) u(t)

SIGNALS & SYSTEMS

Q.13.

Find the Laplace transform of t sin 0 t u (t ) .

(6)

Ans:
L sin (0t) L Using t f(t) - d [F(s)], ds

0 s2 + 02

L [ t sin (0t) u(t) ] = - d ds = 0 - 0(2s) (s2 + 02)2 =


2

0 s2 + 02

2 0 s (s + 02)2

Q.14.

Find the inverse Laplace transform of

s2 s(s + 1)3

(8)

Ans:
F(s) = A= s-2 = A s(s+1)3 s s-2 = -2 3 (s+1) s=0 s-2 = 3 s s = -1 A = -2 F(s) = D=3 + B + C + s+1 (s+1)2 D (s+1)3

A(s+1)3 + Bs(s+1)2 + Cs(s+1) + Ds = s-2 s3 : A+B = 0 B=2 C=2

D=

s2 : 3A + 2B + C = 0

-2 + 2 + 2 + 3 s s+1 (s+1)2 (s+1)3 -t -t -t f(t) = -2 + 2 e + 2 t e + 3 t2 e 2 -t f(t) = [-2 + e ( 3 t2 + 2t + 2 ) ] u(t) 2

27

AE06/AC04/AT04 Q.15.

SIGNALS & SYSTEMS

Show that the difference equation y(n ) y(n 1) = x (n ) + x (n 1) represents an all-pass transfer function. What is (are) the condition(s) on for the system to be stable? (8)

Ans: y(n) y(n-1) = - x(n) + x(n-1)


Y(z) z-1 Y(z) = - X(z) + z-1 X(z) (1- z-1) Y(z) = (- + z-1) X(z) H(z) = Y(z) = - + z-1 = 1- z z- X(z) 1- z-1 Zero : z = 1 Pole : z = Condition for stability of the system : For stability, the pole at z = must be inside the unit circle, i.e. < 1. As poles and zeros have reciprocal values, the transfer function represents an all pass filter system.

Q.16. Give a recursive realization of the transfer function H (z ) = 1 + z 1 + z 2 + z 3 Ans:


H(z) = 1 + z-1 + z-2 + z-3 = 1 z 4 Geometric series of 4 terms 1 1z First term = 1, Common ratio = z 1 As H(z) = Y(z) , we can write X(z) (1 z 1) Y(z) = (1 z 4) X(z) or Y(z) = X(z) (1 z 4) = W (z)(1 z 4) (1 z 1) The realization of the system is shown below.

(6)

Q.17

Determine the z-transform of x 1 (n ) = n u (n ) and x 2 (n ) = n u ( n 1) and indicate their regions of convergence.

(6)

28

AE06/AC04/AT04 Ans:
x1(n) = n u(n) X1(z) = and x2(n) = -n u(-n-1)

SIGNALS & SYSTEMS

1 RoC z-1 < 1 i.e., z > -1 1-z

X2(z) =

- n z-n
n=- n=1

-1

= - -n zn = -( -1z + -2z2 + -3z3 + ) = - -1z ( 1 + -1z + -2z2 + ..) = - -1z 1- -1z = z z- = 1 ; 1 - z-1 RoC

-1 z < 1 i.e., z <

Q.18. Determine the sequence h (n ) whose z-transform is 1 H (z ) = , r <1. 1 2 r cos z 1 + r 2 z 2 Ans:


. H(z) = = 1 , -1 -2 2 1-2r cos z + r z 1 (1-r ej z-1) (1-r e j z -1) A + B j -1 (1-r e z ) (1-r e jz -1) 1 (1-r ej z-1) r <1 , r <1

(6)

r <1

where A=

r e z =1 =
-1

=
-1

1 1- e -j2

B=

1 (1-r e z-1)
j

r e z =1

-j

1 1- ej2

h(n) =

1 ( r ej )n + 1- e-2j
1 e 1 e

1 ( r e-j )n 1- e2j

e j n e jn u(n) h(n) = rn + j 2 j 2
n j(n + 1) j

=r e

- e - j(n + 1)
-j

u (n)

e -e

29

AE06/AC04/AT04
rn sin(n+1) u (n) sin

SIGNALS & SYSTEMS

Q.19.

1 Let the Z- transform of x(n) be X(z).Show that the z-transform of x (-n) is X . z Ans: z x(n) X(z) Let y(n) = x(-n)

-n

(2)

+r

Then Y(z) =

x(-n)z =
n = -

x(r) z
r = -

= x(r) (z ) -1 = X (z )
r = -

-1

-1

2n Q.20. Find the energy content in the signal x (n ) = e n 10 sin . 4 Ans:


x(n) = e 0.1 sin
n

(7)

2n 4
+ +
2

2
-0.2 n

Energy content E =
+

x (n) = e
n=-

sin

n=-

2n 4

E = E =

e-2n e-2n

sin2 n
2

n=- + n=- +

1-cosn
2

= 1 e-2n
2 n=-

[1 (-1) n]

Now 1 (-1) n =

2 for n odd 0 for n even


Also Let n = 2r +1 ; then


E =

e-.2(2r +1 ) =
r=-

e-.4r e- .2
r=-

= e-..2 e-.4r + e.4r


r=0 r=1

The second term in brackets goes to infinity . Hence E is infinite.

Q.21. Sketch the odd part of the signal shown in Fig.

(3)

30

AE06/AC04/AT04

SIGNALS & SYSTEMS

Ans: Odd part xo(t) = x(t) x(-t) 2

Q.22.

A linear system H has an input-output pair as shown in Fig. Determine whether the system is causal and time-invariant.

(4)

Ans: System is non-causal the output y(t) exists at t = 0 when input x(t) starts only at t = +1. System is time-varying the expression for y(t) = [ u (t) u (t-1)(t 1) + u (t 3)
(t 3) u (t-3) ] shows that the system H has time varying parameters.

Q.23. Determine whether the system characterized by the differential equation


d 2 y(t ) dt
2

dy(t ) + 2 y(t ) = x (t ) is stable or not. dt

(4)

Ans: d2y(t) - dy(t) +2y(t) = x(t) dt2 dt L L y(t) Y(s); x(t) X(s); Zero initial conditions
s2 Y(s) sY(s) + 2Y(s) = X(s)

31

AE06/AC04/AT04
System transfer function Y(s) = 1 X(s) s2 s + 2 Hence the system is not stable.

SIGNALS & SYSTEMS


whose poles are in the right half plane.

Q.24 Determine whether the system y(t ) = Ans:


t

x () d is invertible.

(5)

y(t) = x() d
-

Condition for invertibility: H H-1 x(t)

H-1 H = I (Identity operator)

Integration Differentiation

y(t) = H{x(t)}

H-1{y(t)} = H-1 H{x(t)} = x(t) The system is invertible.

Q.25

Find the impulse response of a system characterized by the differential equation y(t ) + a y(t ) = x (t ) .

(5)

Ans:
y(t) + a y(t) = x(t) L x(t) X(s), y(t)

L Y(s), h(t)

L H(s)

sY(s) + aY(s) = X(s), assuming zero initial conditions H(s) = Y(s) = 1 X(s) s+a -at The impulse response of the system is h(t) = e u(t)

Q.26.

Compute the Laplace transform of the signal y(t ) = (1 + 0.5 sin t ) sin 1000t .

(4)

Ans: y(t) = (1 + 0.5 sint) sin1000t = sin 1000t + 0.5 sint sin 1000t
= sin 1000t + 0.5 cos 999t cos 1001t 2 = sin 1000t + 0.25 cos 999t 0.25 cos 1001t 1000 + 0.25 s 2 2 s + 1000 s + 9992
2

Y(s) =

- 0.25 s 2 s + 10012

32

AE06/AC04/AT04

SIGNALS & SYSTEMS

Q.27. Determine Fourier Transform F() of the signal f (t ) = e t cos(t + ) and determine the value of F() . Ans:
We assume f(t) = e-t cos(t + ) u (t) because otherwise FT does not exist FT + f(t) F() = e-t ej(t + ) + e-j(t + ) e-jt dt 2
+

(7)

F() = 1 2

[e-t e-jt ejt + j + e-t e-jt e-jt j] dt

= 1 [e-t + j + e j e ( + 2j)t] dt 2
+ ( + 2 j ) t 1 j e t j e F( ) = e +e 2 ( + 2 j ) 0 0

1 1 j 1 e + e j 2 + 2 j

F() = 1 ( + 2j) ej + e j 2 ( + 2j) = 1 2 cos + 2jej 2 ( + 2j)

= F() 2

cos + j cos j sin ( + 2j)


= 2 cos2+ 2 2 sin + cos

2 (2 + 42) 2 + 2 cos 2 sin2


=

2 (2 + 42)

Q.28. Determine the impulse response h(t) and sketch the magnitude and phase response of the system described by the transfer function
H(s ) =
2 s 2 + o . o 2 2 s + s + o Q

(14)

Ans: H(s) =

s 2 + 0 2 s2 + 0 s + 02 Q

33

AE06/AC04/AT04
H(j) = (j)2 + 02 = (j) + 0 (j) + 02 Q
2

SIGNALS & SYSTEMS


02 - 2 0 - 2 + j 0 Q
2

H(j) =

02 - 2 ( - 2)2 + 2 0 2 Q
2 0

1/ 2

Arg H(j) = - tan-1

0 Q 02 - 2 H(j) Arg H(j) 0 1 0 1 0 00 - /2 0+ 0 + /2


H(j) Magnitude 1

0 arg H(j) + /2 0 - /2

0
Phase

Q.29. Using the convolution sum, determine the output of the digital system shown in Fig. below. Assume that the input sequence is {x (n )} = {3, 1, 3}and that the system is initially at rest. n=0

(5)

Ans: x(n) = { 3, -1, 3 }, system at rest initially (zero initial conditions)


n=0

34

AE06/AC04/AT04
x(n) = 3(n) - (n-1) + 3(n-2) X(z) = 3 - z-1 + 3z-2 Digital system: y(n) = x(n) + 1 y(n-1) 2 Y(z) = X(z) = 3 - z-1 + 3z-2 1 1 z-1 1 1 z-1 2 2 by partial fraction expansion.
n

SIGNALS & SYSTEMS

= -10 -6 z-1 + 13 1 1 z-1 2

1 Hence y(n)= -10 (n) 6 (n-1) + 13 u (n ) 2


Q.30.
Find the z-transform of the digital signal obtained by sampling the analog signal

e 4 t sin 4 t u (t ) at intervals of 0.1 sec.


Ans:
t

(6)

x(t) = e-4 sin 4t u(t), x(n) = x(t z x(n)

T = 0.1 s
n

nT) = x(0.1n) = ( e-0.4 ) sin(0.4n) X(z) z z sin z2 2z cos + 1

= e-0.4 = 0.6703, 1 = 1.4918 = 0.4 rad = 22.92


sin = 0.3894; cos = 0.9211

x(n) = sin n u(n)

n x(n)
X(z) =

X (z/) 1.4918z (0.3894) (1.4918)2 z2 2(1.4918)z(0.9211) + 1 0.5809z 2.2255 z2 2.7482z + 1

X(z) =

Q.31. An LTI system is given by the difference equation y(n ) + 2 y(n 1) + y(n 2 ) = x (n ) . i. Determine the unit impulse response. ii. Determine the response of the system to the input (3, -1, 3).

n=0
Ans: y(n) + 2y(n-1) + y(n-2) = x(n)
Y(z) + 2z-1 Y(z) + z-2 Y(z) = X(z) (1 + 2z-1 + z-2)Y(z) = X(z)

(4)

35

AE06/AC04/AT04
(i). H(z) = Y(z) X(z) = 1 = 1 1 + 2z-1 + z-2 (1 + z-1)2

SIGNALS & SYSTEMS


( Binomial expansion)

= 1- 2z-1 + 3 z-2 - 4 z-3 + 5 z-4 - 6 z-5 + 7 z-6 - .( Binomial expansion) h(n) = (n) - 2(n-1) + 3(n-2) -.. = {1,-2,3,-4,5,-6,7,.} is the impulse response. n=0 (ii). x(n) = { 3,-1,3 } n=0 = 3(n) - (n-1) + 3(n-2) X(z) = 3 - z-1 + 3z-2 Y(z) = X(z).H(z) = 3 - z-1 + 3z-2 = 3(1 + 2z-1 + z-2) 7z-1 1 + 2z-1 + z-2 1 + 2z-1 + z-2 = 3 7 z-1 (1 + z-1)2 y(n) = 3(n) + 7nu(n) is the required response of the system.

Q.32.

The signal x(t) shown below in Fig. is applied to the input of an (i) ideal differentiator. Sketch the responses. x(t) = t u(t) 3t u(t-1) + 2t u(t-1.5) (ii) ideal integrator.

(1+4=5)

36

AE06/AC04/AT04

SIGNALS & SYSTEMS

Ans:
x(t) (i) 0 < t < 1 1 y(t) = t 0 dx(t ) dt 1 1 1.5 (ii) 1 < t < 1.5
t 1

t dt = t2 1 = 0.5 (Nonlinear) 0 2 0

y(t) = y(1) + (3-2t)dt


1 t

= 0.5 + (3t t2) = 0.5 + 3t t2 3 + 1


1 0

= 3t t2 1.5

(Nonlinear)

For t =1: y(1) = 3 1 1.5 = 0.5 (iii) t 1.5 : y(1.5) = 4.5 2.25 1.5 = 0.75 -2

x(t)dt
0.5 0 1 1.5 t

Q.33.

Sketch the even and odd parts of (i) a unit impulse function (iii) a unit ramp function. (ii) a unit step function

(1+2+3=6)

Ans: Even part


Odd part

xe(t) = x(t) + x(-t) 2 xo(t) = x(t) - x(-t) 2

(i) unit impulse function

(ii) unit step function 37

(iii) unit ramp function

AE06/AC04/AT04 Q.34.
t t Sketch the function f (t ) = u sin u sin . T T Ans:

SIGNALS & SYSTEMS


(3)

f(t) 1 .. -T 0 -1 T .. t

f(t) =

1 -1 3

0 < t T, 2T < t 3 T1 T< t 2T, T < t <4T,

Q.35. Under what conditions, will the system characterized by y (n ) =


linear, time-invariant, causal, stable and memoryless?

e
k = no

ak

x(n k ) be

(5)

Ans: y(n) is : linear and time invariant for all k causal if n0 not less than 0. stable if a > 0 memoryless if k = 0 only Q.36.
Let E denote the energy of the signal x (t). What is the energy of the signal x (2t)?

(2)

Ans: Given that


2

E=

x(t )
1

dt
2

To find E =

x(2t )

dt
2

Let 2t =r then E1 =

x(r )

dr 1 = 2 2

x(r ) dr =

E 2

Q.37. x(n), h(n) and y(n) are, respectively, the input signal, unit impulse response and output signal of a linear, time-invariant, causal system and it is given that y(n 2) = x (n n1 ) * h (n n 2 ), where * denotes convolution. Find the possible sets of values of n1 and n 2 . (3) Ans:
y(n-2) = x(n-n1) * h(n-n2) -n -n z-2 Y(z) = z 1 X(z) . z 2 H(z) z-2 H(z) X (z) = z ( n1 + n 2 ) X(z)H (z ) n1+n2 = 2

38

AE06/AC04/AT04

SIGNALS & SYSTEMS

Also, n1, n2 0, as the system is causal. So, the possible sets of values for n1 and n2 are: {n1, n2} = {(0,2),(1,1),(2,0)}

Q.38. Let h(n) be the impulse response of the LTI causal system described by the difference equation y(n ) = a y(n 1) + x (n ) and let h (n ) * h1 (n ) = (n ) . Find h1 (n ) . (4) Ans: y(n) = a y(n-1) + x(n)
Y(z) = az-1 Y(z) + X(z) H(z) = Y(z) = 1 X(z) 1-az-1 H1(z) = 1-az-1 or and and and h(n) * h1(n) = (n) H(z) H1(z) =1 H1(z) = 1 H(z)

h1(n) = (n) a (n-1)

Q.39. Determine the Fourier series expansion of the waveform f (t) shown below in terms of sines and cosines. Sketch the magnitude and phase spectra. (10+2+2=14) Ans:

Define g(t) = f(t) +1. Then the plot of g(t) is as shown , below and,

= 2/2 = 1 because T =2

g(t) =

0 - < t < - /2 2 - /2< t < /2 /2< t < 0

Let g(t) = a0 + (an cos nt + bn sin nt) n=1 Then a0 = average value of f(t) =1

39

AE06/AC04/AT04
an =
2 2
/2

SIGNALS & SYSTEMS


2 cos ntdt

/2

2 sin nt /2 n
-/2

= 2 /n . 2sin n /2

= 4 /n . sin n /2 = 0 if n= 2,4,6 4 /n if n= 1,5,9 if n= 3,7,11 - 4 /n Also, bn =


2 2
/2
/2

2 sin ntdt

4 cos nt /2 n
-/2

= 4 /n [ cos n /2 - cos n /2] = 0

Thus, we have f(t) = -1 + g(t) =


4cost

4cos 3t 4 cos 5t + ....... 3 5

= 4/ { cost cos3t /3 + cos5t/5 ..} spectra : 1


1/3
1/5
1/7

(Magnitude) X /4

-7 -6 -5 -4 -3 -2 -1

2 3

Phase -7 -3 - 3 7

Q.40. Show that if the Fourier Transform (FT) of x (t) is X() , then dx (t ) FT = j X() . dt Ans:
FT x(t) i.e., x(t) = X(j) or X() 1 X(j) ejt d 2 -
+ +

(3)

d [x(t)] = 1 X(j) j ejt d dt 2 - d [x(t)] dt FT j X(j)

1 Q.41. Show, by any method, that FT = ( ) . 2

(2)

40

AE06/AC04/AT04 Ans:
x(t) = 1 X(j) ejt d 2 -
+ +

SIGNALS & SYSTEMS

x(t) = 1 () ejt d = 1 2 - 2 1 2 FT

X(j) = ()

()

Q.42 Find the unit impulse response, h(t), of the system characterized by the relationship :
y(t ) =
t

x ( ) d .

(3)

Ans:
t

y(t) = () d =
-

1, t 0 = u(t) 0, otherwise

Q.43. Using the results of parts (a) and (b), or otherwise, determine the frequency response of the system of part (c). (6)

Ans: As shown in the figure, u(t) = 1/2 + x(t) where x(t) = 0.5, t >0 -0.5, t <0 dx/dt = (t) By (a) FT[ (t)] = jX() X() = 1/j. Also FT[1/2] = () Therefore FT [u(t)] = H(j)= () + 1/j . Q.44. Let X e j denote the Fourier Transform of the signal x (n) shown below .(2+2+3+5+2=14)

( )

41

AE06/AC04/AT04

SIGNALS & SYSTEMS

Without explicitly finding out X e j , find the following :

( )

(i)

X (1)

(ii)

X e j d

( )

(iii) X(-1)

(iv) the sequence y(n) whose Fourier Transform is the real part of X e j .

( )

(v)

X e j

( ) 2 d .
-jn

Ans: j X(e ) =

x(n) e
n = -
j0

(i) X(1) = X(e ) = x(n) = -1 + 1 + 2 + 1 + 1 + 2 + 1 1 = 6


- +
j jn

(ii) x(n) = 1 X(e ) e 2 - (iii) X(-1) = X(e ) = (iv) Real part X(e )
j j

d ; X(e ) d = 2 x(0) = 4
-

x(n) (-1)n = 1+ 0-1+2-1+0-1+2-1+0+1 =2


n = -

xe(n) = x(n) + x(-n) 2 y(n) = xe(n) = 0, n < -7, n > 7 y(7) = 1 x(7) = -1 = y(-7) 2 2 y(6) = 1 x(6) = 0 = y(-6) 2 y(5) = 1 x(5) = 1 = y(-5) 2 2 y(4) = 1 x(4) = 2 = y(-4) 2 y(3) = 1[x(3) + x(-3)] = 0 = y(-3) 2 y(2) = 1[x(2) + x(-2)] = 0 = y(-2) 2 y(1) = 1[y(1) + y(-1)] = 1 = y(-1) 2 y(0) = 1[ y(0) + y(0)] = 2 2 (v) Parsevals theorem:

X(e ) d = 2
-

x(n) = 2(1 + 1 + 4 +1 + 1 + 4 + 1 + 1) = 28
n = -

Q.45

If the z-transform of x (n) is X(z) with ROC denoted by R x , find the

42

AE06/AC04/AT04
z-transform of y(n ) =

SIGNALS & SYSTEMS

x (k ) and its ROC.


k =

(4)

Ans:
z x(n)
n

X(z),

RoC Rx
0

y(n) =

x(k) =
k = -

x(n-k) =
k=

x(n-k)
k=0

Y(z) = X(z)

z-k =
k=0

X(z) , RoC at least Rx ( z > 1) 1 - z-1

Geometric series

Q.46 (i) x (n) is a real right-sided sequence having a z-transform X(z). X(z) has two
poles, one of which is at a e j and two zeros, one of which is at r e j . It is also known that

x (n ) = 1 .

Determine X(z) as a ratio of polynomials in z 1 .

(6)

(ii) If a = 1 , r = 2, = = 4 in part (b) (i), determine the magnitude of X(z) on the 2 unit circle. (4)

Ans:
z . (i) x(n) : real, right-sided sequence X(z) = K (z- re )(z- re ) (z- ae )( z- ae
j -j j -j -j j

X(z) ; x(n) = X(1) = 1

= K z2 zr (e +e ) + r2 j j z2 za (e + e ) + a2 = K 1 2r cos z-1 + r2 z-2 = K. N(z-1) 1 2a cosz-1 + a2 z-2 D(z-1) where K. 1 2r cos + r2 = X(1) = 1 1 2a cos + a2 i.e., K = 1 2a cos + a2 1 2r cos + r2 (ii) a = , r = 2, = = /4 ; K = 1 2().(1/2) + 1 2(2) (1/2) + 4 X(z) = (0.25) . 1 2(2) (1/2) z-1 + 4z-2 1 2().(1/2) z-1 + z-2 = 0.25

43

AE06/AC04/AT04
= (0.25) 1 - 22 z-1 + 4z-2 1 (1/2) z-1 + z-2
j -j

SIGNALS & SYSTEMS


X(e ) =
j

(0.25)

1 - 2 2 e +4e2 -j - j 1 (1/2) e +e2

-j

- j

= - 2 2 + e + 4 e j -j -22+ 4e + e X(e )
j

=1

Q.47 Determine, by any method, the output y(t) of an LTI system whose impulse response h(t) is of the form shown in fig(a). to the periodic excitation x(t) as shown in fig(b). (14)

Ans:

Fig(a)
h(t) = u(t) u(t-1) => H(s) =
1- e -s s

Fig(b)

First period of x(t) , xT(t) = 2t [u(t) u(t- ) ] = 2[ t u(t) (t-1/2) u(t-1/2) 1/2 u(t-1/2)] XT(s) = 2[1/s2 e-s/2 / s2 1/2 e-s/2 / s ] X (s) = XT(s) / 1 e-s/2 Y(s) =
1 es 1 . s 1 e s / 2

1 e 2

s / 2

=
=

2 s
3
2 s3

(1 + es / 2 )[1 es / 2 0.5 s es / 2 ]
(1 e
s3
s

0.5se s / 2 s2

0.5s (e s / 2 + e s ) e s / 2 + e s s2
2

=2

1 e s

1 1 Therefore y(t) = t u(t) (t-1)2 u(t-1) t u t + ( t 1) u ( t 1) 2 2

This gives y (t) = t2 0< t < 1/2 t2 t +1/2 1/2 < t < 1 1/2 t >1

44

AE06/AC04/AT04

SIGNALS & SYSTEMS

(not to scale)

Q.48

Obtain the time function f(t) whose Laplace Transform is F(s ) =

s 2 + 3s + 1

(s + 1)3 (s + 2)2

(14)

Ans:
F(s) = s2+3s+1 = A + B + C + D + E (s+1)3(s+2)2 (s+1) (s+1)2 (s+1)3 (s+2) (s+2)2 A(s+2)2(s+1)2 + B(s+2)2(s+1) + C(s+2)2 + D(s+1)3(s+2) +E(s+1)3 = s2+3s+1 = 1-3+1 = -1 C = s2+3s+1 (s+2)2 s= -1 1 = 4-6+1 = 1 E = s2+3s+1 3 (s+1) s= -2 -1 C = -1

E=1

A(s2+3s+2)2 + B(s2+4s+4)(s+1) + C(s2+4s+4) + D(s3+3s2+3s+1)(s+2) + E(s3+3s2+3s+1) = s2+3s+1 A(s4+6s3+13s2+12s+4) + B(s3+5s2+8s+4) + C(s2+4s+4) + D(s4+5s3+9s2+7s+2) + E(s3+3s2+3s+1) = s2+3s+1 s4 :
3

A+D = 0 ; A+B+1 = 0 as 5(A+D) = 0, E = 1 as 9(A+D) = 0, C = -1, E = 1 as 7(A+D) = 0, C = -1, E = 1

s : 6A+ B+ 5D +E = 0 s2 : 13A+5B+C+9D+3E = 1 s0 : 4A+4B+4C+2D+E = 1

; 4A+5B+1 = 0

s1 : 12A+8B+4C+7D+3E = 3 ; 5A+8B-4 = 0

A+B = -1 ; 4(A+B)+B+1 = 0 or 4+B+1 = 0 or B=3 A=-4 A = -1-3 = - 4 A+D = 0 or D = -A = 4 F(s) = - 4 + 3 + -1 + 4 + 1 (s+1) (s+1)2 (s+1)3 (s+2) (s+2)2 f(t) = L-1[F(s)] = - 4e-t + 3t e-t t2 e-t + 4e-2 t + t e-2t = [e-t(-4 + 3t - t2) + e-2 t(4 + t)] u(t) D=4

45

AE06/AC04/AT04

SIGNALS & SYSTEMS

f(t) = [e-t(-4 + 3t - t2) + e-2 t(4 + t)] u(t)

Q.49 Define the terms variance, co-variance and correlation coefficient as applied to random variables.

(6)

Ans: Variance of a random variable X is defined as the second central moment E[(X-X)]n, n=2, where central moment is the moment of the difference between a random variable X and its mean X i.e.,
+

X = var [X] =
-

(x- X) fx(x) dx

Co-variance of random variables X and Y is defined as the joint moment: XY = cov [XY] = E[{X-E[X]}{Y-E[Y]}] = E[XY]- X Y where X = E[X] and Y = E[Y]. Correlation coefficient XY of X and Y is defined as the co-variance of X and Y normalized w.r.t XY : XY = cov [XY] = XY XY XY Q.50 Determine the total energy of the raised-cosine pulse x(t), shown in Fig.1 defined by: (8) 1 (cos + 1), t x(t) = 2 . 0, otherwise


Fig.1

Ans:
+

Energy E =

( t )dt =

1 3 2 4 (cos t + 1) dt = 4 units.
FT

Q.51 State the sampling theorem, given x ( t ) X () . For the spectrum of the continuous-time signal, shown in Fig.2, consider the three cases f s = 2f x ; f s > 2f x ; f s < 2f x and draw the spectra, indicating aliasing. (8)

46

AE06/AC04/AT04

SIGNALS & SYSTEMS

fx

fx

Fig.2

Ans:
Sampling theorem: Given x ( t ) X () , if X () = 0 for > m , and if s > 2m ,
FT

2 , Ts = Sampling interval, then x(t) is uniquely Ts determined by its samples x ( nTs ) where n = 0, 1, 2, .. ( s > 2 m Nyquist rate.)
where sampling frequency s =

X (f )
f s X ( 0)

-fx

fx

f
fs = f x

-fx

fx (2fx) Guardband

f
f s > 2f x

-fx

fx

2fx

f
f s < 2f x

overlap alia sin g -fx 0 fx f

Q.52

Consider a continuous-time signal x(t).


FT

(8)

(i) (ii)

Show that X ( t ) 2 x () , using duality (or similarity) property of FT s. 1 Find x(t) from X () = , using the convolution property of FTs. (1 + j) 2

Ans: (i)

x(t) =

1 X()e jt d 2 =

Using duality property of FTs, t ,

47

AE06/AC04/AT04
x () = 1 1 X ( t )e jtdt , or , x ( ) = X ( t )e jt dt 2 t = 2 t =
+ jt X( t )e dt , i.e., X(t ) 2x () .
+ +

SIGNALS & SYSTEMS

2 x () =

FT

(ii) Find x(t) from X () =


X () = 1 (1 + j) 2 =

1 (1 + j) 2

, using the convolution property of FTs.

FT 1 1 1 , and, e t u ( t ) . . 1 + j 1 + j 1 + j FT

Convolution property of FTs x ( t ) = x1 ( t ) * x 2 ( t ) X () = X1 ()X 2 () .


+

x(t) =

u ()e ( t ) u ( t )d .

1, 0 < < t > 0 u ()u ( t ) = 0, t < 0.

= t.e t , t > 0 x(t) = t e t u (t ) .

Q.53 Find the difference equation describing the system represented by the block-diagram shown in Fig.3, where D stands for unit delay. (8)

1 2 1 4 Fig.3

Ans: Intermediate variable f(n) between the summers: 1 1 f (n ) = x (n 1) f (n 1) f (n 2) 2 4 y(n ) = 2 x (n ) + f (n ) , or, f (n ) = y(n ) 2 x (n ) f ( n 1) = y( n 1) 2 x ( n 1) f ( n 2) = y ( n 2) 2 x ( n 2) 1 1 1 y(n ) + y(n 1) + y(n 2) = 2 x (n ) + 2 x (n 1) + x (n 2) . 2 4 2 Q.54 For the simple continuous-time RC frequently-selective filter shown in Fig.4, obtain the (8) frequency response H(). Sketch its magnitude and phase for - < < .

48

AE06/AC04/AT04

SIGNALS & SYSTEMS

Fig.4

Ans:

KVL x ( t ) = RC
or,

FT dy( t ) + y( t ) X() = RCjY() + Y() dt Y () 1 1 H () = = = . X () 1 + jCR 1 + j 0

H () =

1 1+ 0
2

arg H() = tan 1 0 magnitude spectrum


3 dB

phase spectrum

0
2

- 0

Q.55

Consider the signal x ( t ) = e t u ( t ) + e 2 t u ( t ) . Express its Laplace Transform in the N (s) form: X (s) = K. , K = system constant. Identify th region of convergence. D(s) Indicate poles and zeros in the s-plane. (8)

Ans:
x ( t ) = e t u ( t ) + e 2 t u ( t ) X (s ) =
L

1 1 + s +1 s + 2

3 s + 2s + 3 N(s) 2 , K = 2. X (s) = =1 =K (s + 1)(s + 2) D(s) s 2 + 3s + 2

49

AE06/AC04/AT04
3 1 Zero s = . 2 2 Poles s = 1,2. ROC R e {s} > 1. R e {s} > 2. Common R o C is R e {s} > 1.

SIGNALS & SYSTEMS

Q.56 Given input x(n) and impulse response h(n), as shown in Fig.5, evaluate (8) y(n ) = x (n ) * h (n ) , using DTFTs.

Fig.5

Ans:

( ) n = 1 n = 0 n =1 ( ) n = 1 n = 0 n =1 2 Y (e j ) = a (e j + 1 + e j ) = a (e j2 + 2e j + 3+ 2e j + e j2 )
H e j = e + j + 1 + e j ; X e j = ae + j + a + ae j
As (n n 0 ) e jn o , y(n) = a(n+2) + 2a(n+1) + 3a(n) + 2a(n-1) + a(n-2). y(n) = {a, 2a, 3a, 2a, a} n=0
DTFT

y(n ) = x (n ) * h (n ) Y (e j ).H (e j ) .

DTFT

Q.57 Determine

Xe

( ) = e j2 5e j + 6 .
j

the

inverse

DTFT,

by

partial

fraction

expansion,

of

(8)

Ans:

3 2 + . j j 1 j 1 j j 2 j e 3 e 2 1 e 1 e e 5e +6 3 2 n n n n 1 1 1 1 x (1n ) = 2 u (n ) + 3 u (n ) = 2 + 3 u (n ) . 3 3 2 2 X e j =

( )

) (

)(

50

AE06/AC04/AT04

SIGNALS & SYSTEMS

Q.58 State the initial-value and final-value theorems of Laplace Transforms. Compute the L 3s + 4 initial-value and final-values for x ( t ) X(s) , where x (s) = . (8) s(s + 1)(s + 2) 2 Ans: Initial-value theorem: If f(t) and its first derivative are Laplace transformable, then
the initial value of f(t) is: f (0 + ) = lim f ( t ) = lim sF(s) .
t 0 + s

Final-value theorem: If f(t) and its first derivative are Laplace transformable, and f(t) is not a periodic function, then the final value of f(t) is: lim f ( t ) = lim sF(s) .
t s 0

4 3 + s Initial value x (0 + ) = lim sX(s) = lim = 0. 1 s s 2 1 + (s + 2 ) s (3s + 4) = 1 . Final value lim x ( t ) = lim sX(s) = lim t s 0 s 0 (s + 1)(s + 2 )2
Q.59 Find, by Laplace Transform method, the output y(t) of the system described by the dy( t ) differential equation: + 5 y( t ) = x ( t ) where input x ( t ) = 3e 2 t u ( t ) and the initial dt condition is y(0) = -2. (8) Ans:
dy( t ) + 5 y( t ) = 3e 2 t u ( t ), y(0) = -2. dt L L 1 y( t ) Y(s), u ( t ), e 2 t . s+2 3 sY (s) y(0 + ) + 5Y (s) = . s+2 2 3 A B 2 + . Y (s) = = + (s + 2)(s + 5) (s + 5) s + 2 s + 5 s + 5 1 3 3 = . A= s = 2 = 1 s+2 s+5 s+5 3 B= y( t ) = e 2 t 3e 5 t u ( t ) s = 5 = 1 . s+2

Q.60 An LTI system is characterised by the difference equation: x(n 2) 9x(n 1) + 18x(n) = 0 with initial conditions x(-1) = 1 and x(-2) = 9. Find x(n) by using z-transform and state the properties of z-transform used in your calculation. (8) Ans:
x(n 2) -9x(n 1) + 18x(n) = 0 By using

x (n n 0 ) z n 0 X (z) + x (n 0 ) + z 1x (n 0 + 1) + z 2 x (n 0 + 2) + ..... + z ( n 0 1) x (1)


51

AE06/AC04/AT04

SIGNALS & SYSTEMS

We get [ x ( 2) + x ( 1) z 1 + z 2 X( z)] 9[ x ( 1) + z 1X ( z)] + 18X ( z) = 0 . 1 2 3 1 2 3 1 2 3


9 1 1

z 1 1 2 . X (z) = = 1 1 1 2 18 1 1 z 1 1 1 z 1 181 z + z 3 6 18 2 n 1 n 1 1 1 1 x (n ) = 2 u (n 1) . 18 16 3
z 1

Q.61 Determine the discrete-time sequence x(n), given that x (n ) X (z) = Ans: Assume that x(n) is casual. Then

z2 + z z 3 3z 2 + 3z 1 (8)

z 3 3z 2 + 3z 1 z 2 + z

z 1 + 4z 2 + 9z 3 + 16z 4 + ......

z 2 3z + 3 z 1 4z 3 + z 1 4z 12 + 12z 1 4z 2 9 11z 1 + 4z 2 9 27 z 1 + 27 z 2 9z 3 16z 1 23z 2 + 9z 3 16z 1 48z 2 + 48z 3 16z 4 25z 2 39z 3 + 16z 4
z n 0 ( n n ) 0 z x ( n ) = ( n 1) + 4(n 2) + 9( n 3) + 16( n 3) + ..... x(n) = {0, 1, 4, 9, 16, ..} n=0

X ( y) = z 1 + 4z 2 + 9z 3 + 16z 4 + .....

Q.62 Explain the meaning of the following terms with respect to random variables/processes: (i) Wide-sense stationary process. (ii) Ergodic process. (iii) White noise. (iv) Cross power spectral density. (8) Ans: (i) Wide-sense stationary process. For stationary processes, means and variances are independent of time, and covariance depends only on the time-difference if in addition, the N-fold joint p.d.f. depends on the time origin, such a random process is called wide- sense stationary process. (ii) Ergodic process. Ergodic process is one in which time and ensemble averages are interchangeable. 52

AE06/AC04/AT04

SIGNALS & SYSTEMS

For ergodic processes, all time and ensemble averages are interchangeable, not just the mean, variance and autocorrelation function. (iii) White noise. White noise is an idealised form of noise, the power spectral density of which is independent of frequency. White is in parlance with white light that contains all frequencies within the visible band of electromagnetic radiation.. (iv) Cross power spectral density. Cross power spectral density of two stationary random processes is defined as the FT of their cross-correlation function

R xy () Sxy (f ), where, R xy () = E{X ( t ).Y( t + )} .


Q.63
A random variable X is characterised by probability density function shown in Fig.6: x 1 , 0 x 2 f x (x) = 2 f X (x ) 0 , otherwise Compute its: Probability distribution function; Probability in the range 0.5< x 1.5; Mean value between 0 x 2; and Mean-square value E(x2). Fig.6 (8) Ans: p.d.f. f X ( x ) =
x2 = f ( ) d 1 d = x , 0< x 2. X 2 4 0
+ x

FT

Probability = f X (1.5) f X (0.5) = (0.5 < X 1.5)


2

1 . 2

2 x Mean value m X = x 1 dx = 2 3 0 2 x Mean-squared value E x = x 2 1 dx = . 3 2 0


2

[ ]
3

Q.64

Determine the fundamental frequency of the signal


x[ n] = e
j 4n

+e

j 3n

.
j 3n 8

Ans:

x (n) = e
Fundamental Frequency =

j 4n 3

+e

=e

j 32n 24

+e

j 9n 24

1 . 24
t3

Q.65

A CT system is described by y (t ) = invariant and stable.

x( )d . Find if the system is time


(6)

Ans: 53

AE06/AC04/AT04
t 3

SIGNALS & SYSTEMS

y(t) =

x( )d .

t t0 3

Let x(t) be shifted by t0 then the corresponding output yi(t) will be yi(t) =

x ( t

t 3

) d =

x( )d where = t
t t0 3

Original output shifted by t0 sec is y0(t) =

x ( ) d

Hence the system is time-invariant. If x(t) is bounded, output will be bounded. Hence the system is stable.
Q.66

Let x(t ) be a real signal and x(t ) = x1 (t ) + x 2 (t ) . Find a condition so that

x(t ) dt =
2

x1 (t ) dt +
2

x (t )
2

dt

(6)

Ans:

x(t ) dt =
2

( x1 (t ) + x2 (t ) ) dt
2

x(t ) dt =

x1 (t ) dt +
2

x 2 (t ) dt + 2 x1 (t ) x 2 (t ) dt
2

The term 2 x1 (t ) x 2 (t ) dt will become zero if x1 (t ) is the even part and x 2 (t ) is the odd part of x(t ) or vice-versa. Then

x(t ) dt =
2

x1 (t ) dt +
2

x (t )
2

dt .

Q.67

If h1 [n] = [n] , h2 [n] = [n 1] + 2 [n 2] , h3 [n] = [n + 1] + 2 [n + 2] are the impulse responses of three LTI systems, determine the impulse response of the system shown in Fig.1.

h1 [n]

+
h3 [n]

Fig. 1

h2 [n]

Ans:

h1(n) = (n) h2(n) = (n-1)+2(n-2) h3(n) = (n+1)+2(n+2) Impulse response of the system, h(n) = h1(n) + h2(n)*h3(n) = (n) + 2(n+1) + 5(n) + 2(n-1) = 6(n) + 2(n+1) + 2(n-1).
54

AE06/AC04/AT04 Q.68

SIGNALS & SYSTEMS

Given that y (t ) = x(t ) h(t ) , determine x(at ) h(at ) in terms of y (t ) . If a is real, for what values of a the system will be (i) causal, (ii) stable? (10)
Ans: y(t) = x(t)*h(t) Thus, Y(s) = X(s)H(s) Now x(at) has Laplace transform (1/a) X(s/a) . Similarly h(at) has Laplace transform (1/a) H(s/a) . Thus Laplace transform of x(at)*h(at) = (1/a2)X(s/a) H(s/a) = (1/a) (1/a) Y(s/a) = Laplace transform of (1/a)y(at) Assuming the original system to be causal and stable, (i) to maintain only causality, a can take any value, (ii) to maintain stability, a > 0.

Q.69

One period of a continuous-time periodic signal x(t ) is as given below.


t , 1 < t < 1 x (t ) = 0, 1 < t < 2

(10)

Determine Fourier series coefficients of x(t ) , assuming its period to be 3.


Ans: x(t ) =
1 a0 = T

t , 1 < t < 1 0, 1 < t < 2

0 1 1 tdt + tdt = 0 1 T

0 1 t 2 t 2 1 + = 2 1 2 0 T

ak =

0 1 1 1 jk t jk t jk t x(t )e 0 dt = te 0 dt + te 0 dt T 0 1 T T

1 T =

jk e jk 0 1 e jk 0 e jk 0 1 e 0 + + + 2 2 2 2 jk jk k k 0 0 0 0

jk jk 0 2 e jk 0 + e jk 0 2 e 0 e 1 + 2 2 2j 2 k 0 k 0 2 sin k 0 cos k 0 1 = + 2 2 T k k 0 0 cos 2 k / T 1 sin 2 k / T = +T k 2 2 k 2 1 a0 T =3 = 3 sin 2 k / 3 cos 2 k / 3 1 a k T =3 = +3 k 2 2 k 2

1 T

Q.70

Determine Fourier series coefficients of the same signal x(t ) as in Q69, but now, assuming its period to be 6. What is the relationship between the coefficients determined in Q69 & Q70? (6)
Ans:Let the period be T2 = 2T1 = 6. Following the above procedure, we get 55

AE06/AC04/AT04
a02 = 1 T2

SIGNALS & SYSTEMS


0 2 2 4 = = 2a01 tdt + tdt = 0 2 T2 T1 0 2 1 ak 2 = te jk 0 t dt + te jk 0 t dt T2 0 2 cos 4 k / T 2 1 sin 4 k / T 2 = 2 + T2 k 4 2 k 2 sin 2 k / T cos 2 k / T 1 1 1 = 2 + T1 = 2ak1 2 2 k 2 k

Thus the Fourier coefficients are doubled when the period is doubled. The function with higher period will have all the harmonics present in the lower period function as even harmonics.
Q.71

The Fourier transform of a signal x(t) is described as 0.5 1 < < 0 , 1 < < 1 and arg X ( j ) = 0.5 0 < < 1 X ( j ) =
0, otherwise

otherwise

Determine whether x(t) is real or complex.


Ans:The magnitude response is symmetric and the phase response is antisymmetric. So x(t ) is real. 4 Determine the inverse Fourier transform of X ( j ) = 2 sin 2 using the

Q.72

convolution property of the Fourier transform.


Ans: X ( j ) =
4 2 2

(4)

sin 2 = sin sin =P(j).P(j) 2 Multiplication in the frequency domain is equivalent to convolution in the time domain. sin Inverse Fourier transform of P(j) = 2 is a pulse
1, p (t ) = 0,

t 1 t > 1.

Therefore
2 t , t 2 x (t ) = p(t ) p (t ) = t >2 0,

Q.73

A system is described by the difference equation y[n] = x[n] + ay[n 1] . Find the impulse response of the inverse of this system. From the impulse response, find the difference equation of the inverse system. (8)
Ans: y[n] = x[n] + ay[n 1]
H ( z) = Y ( z) 1 = X ( z ) 1 az 1

For Inverse System:

56

AE06/AC04/AT04

SIGNALS & SYSTEMS

Q.74

1 = 1 az 1 H ( z) Impulse Response: h1(n) = 1 ( n) a ( n 1) Difference equation: y ( n) = x( n) ax( n 1) Determine the autocorrelation of the sequence { 1, 1, 2, 3} .

Transfer function H 1 ( z ) =

(8)

Ans:x(n) = (1, 1, 2, 3)

Since

rxx (k ) =

x ( m) x ( m k ) = r
m =

xx ( k )

r (0) = x (0) x(0) + x (1) x(1) + x(2) x(2) + x(3) x(3) = 15 r (1) = x (0) x(1) + x (1) x (2) + x( 2) x(3) = 9 = r (1) r ( 2) = x(0) x (2) + x(1) x(3) = 5 = r ( 2) r (3) = x(0) x (3) + x(1) x(3) = 3 = r (3) r ( 4) = 0 r (n) = [3, 5, 9, 15, 9, 5, 3] Determine the cross correlation of the processes x1 (t ) = A cos(2f c t + ) and x 2 (t ) = B sin( 2f c t + ) , where is an independent random variable uniformly distributed over the interval (0,2 ) . (8)
Ans:x1(t) = A cos (2fct + ) and x2(t) = B sin (2fct + ) Rxx() = E{A cos {2fc(t + )+}B sin (2fct + )}

Thus

Q.75

AB E[sin {2fc(-)} + sin {2fc(2t + + 2)}] 2 AB AB 1 =sin(2fc) + sin{2fc(2t ++2)}d 2 2 2

=-

AB sin (2fc) 2

Q.76

sin t n is sampled by p (t ) = t . Determine and t 2 n = (8) sketch the sampled signal and its Fourier transform.

A signal x(t ) =

Ans: x(t ) =

sin t n sampled by p (t ) = t . Thus, the sampled signal t 2 n =

is
n sin 2 . x (n) = n 2

DTFT of x( n) is a pulse

57

AE06/AC04/AT04

SIGNALS & SYSTEMS

2, X ( ) = 2 0 , otherwise
Q.77

Determine the Fourier transforms of (i) x1[n ] = sin n0 and (ii) x2 [n] = (sin n0 )u[n ]
Ans: (i)
x1 [n ] = sin n 0 = e j0 n e j0 n 1 j0 n = e e j0 n 2j 2j 1 [ ( + 0 ) ( 0 )] X ( ) = 2j

(8)

(ii)

x2 [n] = (sin n0 )u ( n) =

e j 0 nu ( n) e j 0 nu ( n) 2j 1 1 1 X ( ) = j0 j j0 j 2 j 1 e e 1 e e 1 j 1 1 = e j j j0 j0 2j e e e e
sin 0 = e j j j 2 2e cos 0 1 + e

sin 0 1 2 cos cos 0

Q.78

Find the inverse Laplace transform of X (s ) = possible ROCs.


Ans:

s 4 + 3s 3 4 s 2 + 5s + 5 for all s 2 + 3s 4 (8)

H(s) =

s 4 + 3s 3 4 s 2 + 5 s + 5 s 2 + 3s 4 5s + 5 5s + 5 3 2 = s2 + = s2 + . = s2 + 2 + ( s + 4)( s 1) s + 3s 4 s + 4 s 1 h (t ) = d 2 (t ) + 3e 4t u (t ) + 2e t u (t ), ROC > 1 dt 2 d 2 (t ) h (t ) = + 3e 4t u (t ) 2e t u (t ), ROC 4 < <1 dt 2 d 2 (t ) h (t ) = 3e 4t u (t ) 2e t u ( t ), ROC < 4 2 dt

Q.79

Using Laplace transform, find the forced and natural responses of the system d 2 y (t ) dy (t ) dx(t ) described by +5 + 6 y (t ) = + 6 x(t ) when the input is a unit 2 dt dt dt step function and the initial conditions of the system are y (0 + ) = 1 and
y ' (0 + ) = 2.

(8)

Ans:Taking the unilateral Laplace transform of both sides of the given eqn, we obtain 58

AE06/AC04/AT04

SIGNALS & SYSTEMS


( s 2 + 5s + 6)Y ( s ) sy (0 + ) y ' (0 + ) 5 y (0 + ) = ( s + 6) X ( s) x(0 + )

Solving for Y(s)


Y ( s) = ( s + 6) X ( s) x(0 + ) sy (0 + ) + y ' (0 + ) + 5 y (0 + ) + ( s + 2)(s + 3) ( s + 2)( s + 3)

The first term is associated with the forced response of the system, yF(t). The second term corresponds to the natural response, yN(t). Substituting for X(s) = 1/s, x(0+) = 1, y (0 + ) = 1 , y ' (0 + ) = 2 , we obtain
6 s+7 Y ( s) = s( s + 2)(s + 3) + ( s + 2)( s + 3) 3 2 5 4 1 Y ( s) = + + s s + 2 s + 3 s + 2 s + 3

y (t ) = 1 3e 2t + 2e 3t u (t ) +(5 e 2t -4 e 3t ) u (t )

Thus,

y F (t ) = 1 3e 2t + 2e 3t u (t ) ,

y N (t ) = (5 e 2t -4 e 3t ) u (t ) .

Q.80

1 + z 1 . For what values 1 az 1 1 bz 1 of a and b will the system be (i) unstable, (ii) non-causal? (8)

A casual system is described by H ( z ) =

)(

Ans: H ( z ) =

z ( z + 1) ( z a )( z b)

The poles are z = a, b. (i) Causal: both a and b 1. Stable: both a and b < 1. (ii) Unstable: both or either a or b 1. Non-causal: both or either a or b > 1.
Q.81

Determine the ROC of aX ( z ) + bY ( z ) , given that

0.25 z , z > 0.5 . (z 0.5)(z 1.5) (z 0.25)(z 0.5) For what relationship between a and b the ROC will be the largest? (8)
X (z ) = z , 0.5 < z < 1.5, Y ( z ) =

Ans:X(z) =

z , 0.5<z< 1.5 ( z 0.5)( z 1.5) 0.25 z Y(z) = , z> 0.5 ( z 0.25)( z 0.5)
0.25a + 0.375b z (a + 0.25b) z a + 0.25b aX(z) + bY(z) = ( z 0.5)( z 1.5)( z 0.25)

Since ROC is decided by the three poles, ROC of aX(z) + bY(z) is 0.5<z< 1.5. However, there is a possibility of cancellation of one of the poles by the created zero. If the pole at 0.5 can be cancelled, then we shall have the ROC given by 0.25 < |z| < 1.5 the maximum stretch. The condition is
0.25a + 0.375b = 0.5 , i.e., a = b. a + 0.25b

Q.82

Find whether the function y(t) = x(t).cos(100t) represent a Linear, Causal, time invariant system. (8)
59

AE06/AC04/AT04

SIGNALS & SYSTEMS

Ans: y (t ) = x(t ) cos100t If the inputs are x1 (t ) and x 2 (t ) , then the corresponding outputs are

y1 (t ) = x1 (t ) cos100t y 2 (t ) = x 2 (t ) cos100t .
Now if the input x3 (t ) is a linear combination of x1 (t ) and x 2 (t ) , i.e., x3 (t ) = ax1 (t ) + bx 2 (t ) where a and b are arbitrary scalars. Then y 3 (t ) = x3 (t ) cos100t = [ax1 (t ) + bx 2 (t ) ]cos100t = ax1 (t ) cos100t + bx 2 (t ) cos100t = ay1 (t ) + by 2 (t ) . Thus, we conclude that the system is linear. Since the response depends only on the present values, the system is causal. y ( t ) x ( t ) cos 100 (t ) y (t ) , the system is time varying. Since =
Q.83

Find the even and odd parts of the following functions (i) f (t ) = t sin t (ii) f (t ) = a0 + a1t + a2t 2
Ans:

(4)

f o (t ) = (i) Here

1 [ f (t ) f (t )] f e (t ) = 1 [ f (t ) + f (t )] 2 2 , f (t ) = t sin t f o (t ) = 1 [t sin t (t ) sin(t )] = 0 2

f e (t ) =

1 [t sin t + (t ) sin(t )] = t sin t 2

(ii) Here f e (t ) =
Q.84

f (t ) = a + a1 (t ) + a 2 t 2

f o (t ) =

1 {a + a1 (t ) + a2 t 2 } {a + a1 (t ) + a2 (t ) 2 } = a1t 2

1 {a + a1 (t ) + a2 t 2 }+ {a + a1 (t ) + a2 (t ) 2 } = a0 + a 2 t 2 2
(4)

Find the average power of the signal x(t ) = (e 5t + 1) u (t ) .


Ans:Average power over an infinite interval
P = Lt T = Lt T = 1 2 W 1 2T
T

x (t )
10t

dt
5 t

1 2T

(e

+ 1 + 2e

)u(t )dt = 1 2T

(e
0

10t

+ 1 + 2e 5t dt

60

AE06/AC04/AT04

SIGNALS & SYSTEMS

Q.85

Find the Fourier Series of the following periodic wave form and hence draw the spectrum. (8)

f (t ) = A cos
Ans:The function

t,

2 < t < 2 has even symmetry. T 1 Fn (t ) = f (t )e jn0t dt T 0


= 1 jn t A cos t e 2 dt 4 2 4
2

j t 2 j t A e 4 +e 4 = 4 2 2

t jn 2 dt e

2 j t (1 2 n ) 2 j 4 t (1 2 n ) e 4 A e = + 8 j (1 2n ) j (1 2n ) 4 4 2 2 sin (1 2n) sin (1 + 2n) A 2 2 = + (1 + 2n ) (1 2n ) 2A 2A 8A F0 = F1 = F2 = , 3 , 15

Q.86

Find the trigonometric Fourier series of the following wave form.

(8)

Ans:Here T = 1, 0 = 2 , f (t ) = 2Vt .

Since the function exhibits an odd symmetry, a o = a n = 0 . T /2 4 bn = f (t ) sin n 0 tdt T 0

61

AE06/AC04/AT04

SIGNALS & SYSTEMS

= 4 (2Vt ) sin 2ntdt


0

0.5

t ( cos 2nt ) 0.5 0.5 ( cos 2nt ) = 8V 1. dt 2n 2n 0 0


t ( cos 2nt ) 0.5 ( sin 2nt ) = 8V . 2n 2n.2n 0 2V = cos n + 0 n Now 2V f (t ) = bn sin 2nt = cos n. sin 2nt
1 0.5

2V 1 1 sin 2t sin 4t + sin 6t + .... 2 3

Q.87

Define signum and unit step functions? Find the Fourier transforms of (8) these functions.
Ans:Signum function
t >0 1, Sgm (t ) = 1, t < 0

F ( ) = Lt a 0 e

a t

Sgm(t )e jt dt

FT [Sgm] 0 = Lt a 0 e ( a + j )t dt + e ( a + j )t dt 0 1 1 2 = + = j j j 2 F ( ) = Unit Step Function

1 t 0 u (t ) = 0 t < 0

Since
u (t ) =

Sgn (t) = 2u (t ) 1 ,
1 [sgn(t ) + 1] 2 1 1 2 FT of u (t ) = + 2 ( ) = + ( ) 2 j j

Q.88

Determine the Fourier transform a two-sided exponential function t (8) x(t ) = e and draw its magnitude spectrum.
Ans:

f (t ) = e

62

AE06/AC04/AT04

SIGNALS & SYSTEMS

F ( ) =

jt

dt =

(1 j ) t

+ e (1+ j )t dt
0

1 1 2 = + = 1 j 1 + j 1 + 2 2 F ( ) = 1+ 2
Q.89

ind the Discrete Fourier transform of the following sequences. (i) x(n ) = a n , 0 < a < 1 (Find N point DFT) (Find 4 point DFT) (ii) x(n ) = cos n
4

(8)

Ans:
X (k ) =

x ( n) e
n =0

N 1

j 2nk / N

(i)
= =

k = 0,1,..., N 1

a e
n=0

N 1

n j 2nk / N

(ae
n=0

N 1

j 2nk / N n

1 ae j 2k / N 1 ae j 2k / N

=
(ii) N = 4 x(n) = cos 0, = 1,
N 1 n =0

1 aN , 1 ae j 2k / N

k = 0, 1, ...., N 1

cos

4 0707,

cos 0,
,

3 4 -0,707 cos

x ( n) e X (k ) =
X (k ) = {X (0), ={ 1,

j 2nk / N

k = 0,1,..., N 1
j 2nk / N

x ( n) e
=
n =0

k = 0, 1, 2, 3 X (3)} 1 + j1.414}

X (1), 1 j1.414,

X (2), 1,

Q.90

(i) Find the circular convolution of the following sequence (rectangular)


1, 0 n N 1 x1 (n) = x 2 ( n) = 0, otherwise

(ii) Compute the DFT of a) x (n ) = (n ) b) x(n ) = (n n 0 )


X 1 ( k ) = X 2 (k ) =

(4)

W
n =0

N 1

nK N

Ans: (i)

N , k = 0 = 0therwise 0

63

AE06/AC04/AT04
N 2 , k = 0 elsewhere X 3 ( k ) = X 1 ( k ) X 2 (k ) = 0,

SIGNALS & SYSTEMS

(ii) a)

x3 ( n ) = N , 0 n N 1 X ( k ) = DFT [ ( n)] = 1

jno X ( j ), we get b) Since x[n no ] e jmo DFT [ (n n0 ] = e

Q.91

Find the Nyquist frequency of the following signals. (i) Sa (100 t ) (iii) 25 cos(500 t )
x=

(ii) Sa 2 (100 t ) (iv) 10 sinc(2t )

(8)

Ans: Sa (i) This function will have frequency response a rectangular pulse with 100 100 fm = , fs = 2 fm = 2 Hz maximum frequency (ii) The sin2(2ft) = (1-cos 4ft)/2, the maximum frequency will be 2f. 200 100 fm = , fs = 2 fm = Hz 500 fm = , f s = 2 f m = 500 Hz 2 (iii) sin t sin 2t sinc t = . 10 2t . Its frequency response will t (iv) Thus 10 sinc(2t) =

sin x x

be a rectangular pulse X ( f ) such that the maximum frequency f m = 1 Hz. Hence sampling frequency f s = 2 f m = 2 Hz.
Q.92

Define ideal low pass filter and show that it is non-causal by finding its (8) impulse response.
Ans:
j 2ft 0 , B f B e H( f ) = 0, f > B B B e j 2f (t t0 ) j 2f ( t t 0 ) = h(t ) = B e df j 2f (t t 0 ) B B

1 e j 2B (t t0 ) e j 2B (t t0 ) j 2 (t t 0 ) sin 2B (t t 0 ) = 2B = 2 BSinc[2 B (t t 0 )] 2B (t t 0 ) =
The system is non-causal as there is some response before t = 0.
Q.93

Obtain the Laplace transform of the square wave shown.


64

(8)

AE06/AC04/AT04

SIGNALS & SYSTEMS

Ans:

[ f (t )] = f (t )e

0
2T

2T

st

dt + f (t )e dt +.... +
st

4T

( n + 2 )T

f (t )e

st

dt

2T
st

nT

2T 2 sT 2T nsT st st = f (t )e dt + f (t )e dt .e + ... + f (t )e dt .e 0 0 0 2T = 1 + e 2 sT + e 4 sT + .... + e nsT f (t )e st dt 0 2T T 2T 1 1 st st st = f ( t ) e dt = 0 . 5 e dt + 0 . 5 e dt 2 sT 1 e 2 sT 0 T = 1 e 0 1 0 . 5 0 . 5 = e sT 1 + e 2 sT e sT s s 1 e 2 sT 2 1 0 . 5 (1 e sT ) = 2 sT s 1 e sT 0.5 1 e = s 1 + e sT

Q.94

Find the inverse Laplace transforms of the following functions. (i)


s2

(s + a )

+b

(ii) ln s2

s +1 s+ 2

(8)

Ans:
1

(i)

(s + a )

s+a a b = s 2 2 2 2 b ( s + a) + b +b (s + a) + b
2

a d f (t ) = e at cos bt e at sin bt u (t ) f (t ) + f (0 + ) b = dt where

a 2 at at at at be sin bt ae cos bt + ae cos bt + e sin bt + (t )u (t ) b = 2 a at = b sin bt 2a cos bt b sin tbt e u (t ) + (t ) s +1 = F ( s ) = (ii) ln s + 2 ln ( s + 1) ln ( s + 2)

65

AE06/AC04/AT04
dF 1 1 = ds s + 1 s + 2 dF = e t e 2t u (t ) ds

SIGNALS & SYSTEMS

Hence,
1

[F (s)] = 1

dF t ds 1 e t e 2t u (t ) = t
1

Q.95

Obtain the z transforms and hence the regions of convergence of the following sequences. (i) x(n ) = [u (n ) u (n 10 )] 2 n (ii) x (n ) = cos ( n ) u (n ) (8)
Ans: (i)
x (n) = [u (n) u (n 10)]2 n

Z [x( n)] = 2 n Z n
n=0

(ii)
Z [x (n)] =

1 1 2z n = 9 1 1 1 = z > 1 2 n =0 2 z 2 z , ROC is e jn + e jn x(n) = cos(n )u (n) = u ( n) 2


9 1 9 jn n 1 j 1 jn n e z + e z = e z 2 n=0 n =0 2 n =0

) + (e
n n =0

n z 1

1 1 1 + j 1 j 1 2 1 e z 1 e z 1 1 1 z = + = 1 1 2 1 + z 1+ z z +1,

ROC is

z >1

Q.96

A second order discrete time system is characterized by the difference equation y (n ) 0.1 y (n-1)-0.02 y (n-2 ) = 2 x(n ) x(n 1) . Find y (n ) for n 0 when x(n) = u(n) and the initial conditions are given as y(1) = 10, y(2) = 20 (8)
Ans:
y ( n) 0.1 y ( n 1) 0.02( n 2) = 2 x( n) x( n 1) ,

Since

Y (z ) 0.1 Y (1) + z 1Y (z) 0.02 Y (2) + z 1Y(1) + z 2 Y (z) = 2X ( z ) z X ( z ) Substituting the initial values and rearranging, we get
Y ( z ) 1 0.1z 1 0.02 z 2 + 1 0.4 + 0.2 z 1 =
66
1

2z 1 z 1 z 1

AE06/AC04/AT04
Y ( z) =

SIGNALS & SYSTEMS


( z 0.2)( z 0.1) 2 z 1 0.2 = 0.6 z 1 z z2 (2 z 1) z 2 0.2 0.6 Y ( z) = ( z 1)( z 0.2)( z 0.1) z ( z 0.2)( z 0.1) ( z 0.2)( z 0.1) = Y1 ( z ) + Y2 ( z ) + Y3 ( z )
Y1 ( z ) ( 2 z 1) z = z ( z 1)( z 0.2)( z + 0.1)

Now

By partial fraction expansion, we get 1.13 z 0.5 z 0.36 z Y1 ( z ) = + + ( z 1) ( z 0.2) ( z + 0.1) Similarly 2 z 2 z Y2 ( z ) = + 3 ( z 0.2) 3 ( z + 0.1)

Y3 ( z ) = 0.4
Thus,

z z + 0.2 ( z 0.2) ( z + 0.1)

Y (z) =
Hence

1.13 z 0.56 z 0.83 z + ( z 1) ( z 0.2) ( z + 0.1)


n n

y (n) = 1.13 0.56(0.2) + 0.83(0.1) u ( n)

Q.97

A continuous random variable has a pdf f (x ) = Kx 2e x ; x 0 . Find K, and mean and variance of the random variable. (8)
Ans: By the property of PDF

Kx
0

2 x

e dx = 1

Or, 2K = 1 K = Mean value of x is


E ( x) = Now
E(x 2 ) =
2 x 3 x xf ( x)dx = xKx e dx = 0.5 x e dx = 3 0
Rx

x
Rx

f ( x )dx =

x 2 Kx 2 e x dx =0.5

x 4 e x dx = 12

Variance of x is 2 V ( x) = E ( x 2 ) {E ( x)} = 12 - 9 = 3.

Q.98

Find the autocorrelation of the following functions: (i) g (t ) = e at u (t ) (ii) g(t) = A where A is a rectangular pulse with period T and
67
t T t T

AE06/AC04/AT04

SIGNALS & SYSTEMS (8)


g (t ) = e at u (t )

magnitude A.
Ans:

(i)
R g ( ) =

g (t ) g (t )dt

e at e a ( t )u (t )dt = e a

e 2 at dt =

e a 2a

(ii)
R g ( ) =

t g (t ) = A T

g (t ) g (t )dt

R g ( )

For < T
R g ( )

A 2 dt = A 2 (T )

For T
R g ( ) g (t ) g (t )dt 0, = =
T

2 A (T ), < T = T. 0

Q.99

Find the Fourier series of the following periodic impulse train.

(8)

Ans:
A0 = I T0 T0 / 2 2nt 2I T0 / 2 (t ) cos T0 dt = T0 T0 / 2 2nt T0 / 2 (t ) sin T0 dt =0
1 T0

T0 / 2

T0 / 2

(t ) dt =

2I T0 2I Bn = T0
An =

I 2I x(t ) = + T0 T0

2nt I = cos T0 T0 n =1

e
n =

2nt T0

Q.100

The Magnitude and phase of the Fourier Transform of a signal x(t) are shown in the following figure. Find the signal x(t).

Ans: 68

AE06/AC04/AT04
X ( j ) = , W W <0 / 2, ( j ) = X ( j ) = / 2, > 0
e j / 2 , W 0 X ( j ) = j / 2 , 0 W e 1 2

SIGNALS & SYSTEMS

and Thus

x(t ) = F 1 [X ( j )] =
=

X ( j )e jt d

W 1 0 j / 2 jt e e d + e j / 2 e jt d 0 W 2 W W jt ) 1 = e j ( / 2t ) d + e j ( / 2 d 0 0 2

1 2

W jt ) W e j ( / 2t ) + e j ( / 2 d = cos( / 2 t )d = (sin t )d 0 0

2 1 1 Wt W t Wt = [1 cos Wt ] = 2 sin 2 = 2 sinc 2 t t 2 2 2

Q.101

Find the Discrete Time Fourier Transforms of the following signals and draw (8) its spectra. (i) x1 (n ) = a
n

a <1

(ii) x 2 (n ) = cos 0 n where 0 =


Ans:
(i) x(n) = a , a < 1
X e j =
n

2 . 5

( ) a
n =

e j n =

a
n =

e jn +

a
n=0

e j n =

(ae ) + (ae )
j
n j n =1 n=0

ae j 1 1 a2 = + = 1 ae j 1 ae j 1 2a cos + a 2 1 (ii) x(n) = cos 0 n = e j0 n + e j0 n 2 2 2 X (e j ) = 2l + + 2l 5 5 l = l =

2 2 = + + , 5 5

<

Q.102

The frequency response for a causal and stable continuous time LTI system 1 j is expressed as H ( j ) = . (8) 1 + j (i) Determine the magnitude of H ( j ) (ii) Find phase response of H ( j ) (iii) Find Group delay.
Ans:

69

AE06/AC04/AT04

SIGNALS & SYSTEMS

(i) (ii) (iii)


Q.103

=1 1+ 2 H ( j ) = tan 1 ( ) tan 1 ( ) = 2 tan 1 ( ) d d 2 Group delay = ( 2 tan 1 ) = H ( j ) = d d 1+ 2 1 + j

[H ( )] = 1 j

1+ 2

Find the Nyquist rate and Nyquist interval for the continuous-time signal 1 (4) x (t ) = cos(4000t ) cos(1000t ) . 2 Ans:
1 1 [cos 5000t + cos 3000t ] cos 4000t. cos1000t = 2 4 The highest frequency present is h = 5000 , i.e., fh = 2.5 kHz. Nyquist rate is x(t ) =

5 kHz and Nyquist interval = 1/5 k = 0.2 msec.


Q.104

Consider a discrete-time LTI system with impulse response h (n ) given by h(n ) = n u (n ) . Determine whether the system is causal, and the condition for stability. (4)
Ans: Since h(n) = 0 for n < 0, the system is causal. Now

h( k ) = k u ( n) =
k = k = k =0

1 , < 1. 1

Thus the system is stable if < 1 .


Q.105

Check for Causality, Linearity of the following signals. (i) y (t ) = x t (ii) y (t ) = x(t 2 )

( )

(8)

(iii) y (t ) = 10 x(t + 2 ) + 5

(iv) y[n] =

x[n]
k =

Ans: (i) Non-causal: y (0.01) = x(0.1), i.e., y depends upon the future value of x. linear: it is of the form y = mx. (ii) Non-causal: y ( 2) = x(4), i.e., y depends upon the future value of x. linear: it is of the form y = mx. (iii) Non-causal: y(t) depends upon x(t+2) the future value of x(t). Non-linear: it is of the form y = mx + c. (iv) Non-causal: It has the value for n < 0. Linear : it is of the form y = mx. Q.106

Determine the Laplace transform of the following functions. (i) x(t ) = cos 3 (3t ) (ii) x(t ) = t sin at Ans: (i) 1 x(t ) = cos 3 t = [cos 9t + 3 cos 3t ] 4 70

(6)

AE06/AC04/AT04
x(t ) =

SIGNALS & SYSTEMS


1 s 3s 1 s 3s . + 2 = 2 + 2 2 2 2 4 s + 9 s + 3 4 s + 81 s + 9 x(t ) = t sin at d a 2as d x(t ) = (sin at ) = 2 = 2 . 2 ds ds s + a ( s + a 2 ) 2

(ii)

Q.107

2 1 + s+3 s2 Determine the impulse responses if the system is (i) stable (ii) causal. State (10) whether the system will be stable and causal simultaneously. The transfer function of the system is given by H (s ) =
Ans:There are following three possible impulse responses. h1 (t ) = 2e 3t u (t ) + e 2t u (t ), > 2 h2 (t ) = 2e 3t u (t ) e 2t u ( t ), - 3 < < 2 h3 (t ) = 2e 3t u ( t ) e 2t u (t ), < 3 h1(t) is causal but unstable due to the pole at s = 2, h2(t) is non-causal due to the pole at s = 2 but stable, and h3(t) is also non-causal due to both the poles but stable. Thus, the system cannot be both causal and stable simultaneously.

Q.108

Determine the inverse Z transform of the following X(z) by the partial fraction expansion method. (8)
X ( z) = z+2 2z 2 7 z + 3

if the ROCs are (i) z > 3 , (ii) z <


z+2 2z 7z + 3
2

1 1 , (iii) < z < 3 2 2

Ans: X ( z ) =

X ( z) z+2 z+2 2/3 1 1/ 3 = = = + 2 z z z 0.5 z 3 2 z z 7 z + 3 2 z (z 0.5)( z 3) 2 z (1 / 3) z Or X ( z ) = , poles are p1 = 0.5, p 2 = 3 + 3 z 0.5 z 3

(i) When z > 3 all poles are interior, x(n) is causal. Therefore, (ii) When z < 2 1 1 x(n) = (n) u (n) + (3) n u (n) 3 3 2 1 , all poles are exterior, x(n) is non-causal. 2
2 1 1 x(n) = (n) + u (n 1) (3) n u (n 1) 3 3 2
n n

(iii) When

1 < z < 3, p1 is interior and p 2 is exterior. 2

2 1 1 x(n) = (n) u (n) (3) n u ( n 1) 3 3 2


Q.109

A Causal discrete-time LTI system is described by 71

AE06/AC04/AT04
y(n )

SIGNALS & SYSTEMS

3 1 y (n 1) + y (n 2 ) = x(n ) 4 8 where x(n ) and y (n ) are the input and output of the system, respectively. (i) Determine the H(z) for causal system function (ii) Find the impulse response h(n) of the system (8) (iii) Find the step response of the system

Ans:

i)

(ii)

3 1 1 z Y ( z ) + z 2Y ( z ) = X ( z ) 4 8 Y ( z) 1 z2 1 = H ( z) = = ,z > . 3 1 1 1 X ( z) 2 1 z 1 + z 2 z z 4 8 2 4 H ( z) z 2 1 = = 1 1 1 1 z z z z z 2 4 2 4
Y (z)

1 n 1 n h(n) = 2 u (n) 2 4
(iii) Here
z , z >1 z 1 z3 Y ( z) = X ( z)H ( z) = , z >1 1 1 ( z 1)( z )( z ) 2 4 8 z 1 z z = 2 + , z >1 1 3 1 3 z 1 z z 2 4 n n 8 1 11 y (n) = 2 + u (n) 3 4 3 2 X ( z) =

Q.110

A random variable X has the uniform distribution given by 1 , for 0 x 2 f X ( x ) = 2 otherwise 0, Determine mean, mean square, Variance.

(10)

Ans:

Mean m x =

xf x ( x)dx =

x 2 dx =
0

Mean square X 2 == E ( X 2 ) = Variance:

2 x f x ( x)dx =

1 2

x
0

4 dx = 2 3

x 2 = E( X 2 ) mx 2

4 1 = 2 2 = 2 3 3

72

AE06/AC04/AT04

SIGNALS & SYSTEMS

x =
Q.111

Discuss the properties of Gaussian PDF.

(6)

Ans: Property 1: The peak value occurs at x = m, i.e., mean value 1 at x = m (mean) f x ( x) = 2 Property 2: Plot of Gaussian PDF exhibits even symmetry around mean value, i.e., f x (m ) = f x ( m + )

Property 3: The mean under PDF is 1 / 2 for all values of x below mean value and for all values of above mean value, i.e., 1 P ( X m) = P ( X > m) = 2
Q.112

A stationary random variable x(t) has the following autocorrelation function

R x ( ) = 2 e

filter whose impulse response is h( ) = e u ( ) where is a constant, u ( ) is unit step function. (i) Find power spectral density of random signal x(t). (ii) Find power spectral density of output signal y(t). (8)
Ans: (i)
S X ( ) = FT[RX (t )] = =

where 2 , are constants. Rx (t ) is passed through a

R X ( )e j d

2e e j d =

2 2 2 +2

(ii)

H ( ) = FT[h( )] = S y ( ) = H ( ) S X ( ) =
2

e u ( )e j d =
2

+ j

2 2 S X ( ) = 2 + j +2 2 +2

Q.113

Determine the convolution of the following two continuous time functions. (8) x(t ) = e at u (t ) , a > 0 and h(t ) = u (t ) Ans:
y (t ) = h(t ) * u (t ) =

x ( ) h(t ) d =

e a u ( )u (t )d =

e
o

1 a 1 1 e = 1 e at . 0 a a

[ ]

Q.114

Determine signal energy and power of the following signals (i) x(n ) = u (n ) (ii) x(t ) = e 3t
Ans:

(8)

73

AE06/AC04/AT04

SIGNALS & SYSTEMS


E=

(i)
P = lt N
T

x 2 [n] =

u [n] = 1
2
n =

1 2N
3t 2

n = N 2

x [n ] = lt N
T

n= N

1 2N

u [n] = 1.
2
n= N

(ii)

E=

1 [e ] dt = [e ]dt = 6 [e
6 t T

6T

e 6T =

P = LtT

1 1 6T E = LtT e e 6T = 2T 12T

Q.115

Find the DTFT of the sequence x(n) = u(n).


Ans:

(4)

DTFT x(n) = X (e j ) = It is not convergent for = 0.

x(n)e jn = 1.e jn =
n = n =0

1 1 e j

X e j =

( )

e j / 2 2 sin

, 0.

Q.116

Find the inverse Fourier transform of ( ) . Ans: 1 1 j t 1 F [ ( )] = e ( )e jt d = 2 2

(4)

[ ]

=0

1 2

Q.117 Check whether the following signals are energy or power signal and hence find the corresponding energy or power. (6) (t ) (i) x(t ) = Ae u (t ), > 0 2 (ii) x(t ) = cos 0 t Ans: (i) x(t ) = Ae t u (t ), > 0

Then

E=

x(t ) dt = A2

2 e 2t A = 2 0 2

Since 0 < E < , x(t ) is an energy signal. (ii) Since x(t ) = cos 2 o t is a periodic function, it is a power signal.
P = Lt To 1 To
To / 2

x 2 (t )dt

To / 2

1 = Lt T To

To / 2

To

1 cos ot dt = Lt T To /2
2 2 o o

To / 2

[cos

ot dt

To / 2

= Lt T

1 To

To / 2

To / 2

3 1 8 [3 + 4 cos 2 t + cos t ]dt = 8


74

AE06/AC04/AT04 Q.118

SIGNALS & SYSTEMS (10)

Find the convolution of two rectangular pulse signals shown below.

Ans: For t 4 and t 10 the output is 0.

For For For

4 t 6 , y (t ) = 2dt = 2(t 4)
4

6 < t 8 , y (t ) = 2dt = 2(8 6) = 4


6

8 < t 10 , y (t ) = 2dt = 2(10 t )


t

10

Thus y(t) is as shown in the figure.


Q 119 Given the Gaussian pulse x(t ) = e t , determine its Fourier transform. Ans:
X ( ) =
Expressing
2

(8)

x(t )e jt dt =

e t e jt dt =
2

e (t

+ j t )

dt

t 2 + jt = t +

j 2

2 4

we have
X ( ) =

t + e

j 2

2 2 e 4 dt = e 4
j 2
,

t + e

j 2

dt

Let

u = t +

Then

2 2 X ( ) = e 4

(u )2

2 2 f 2 du = e 4 =e 2

Q.120 Find the exponential Fourier series of the following signal.

(8)

75

AE06/AC04/AT04 Ans: Here T = 1sec, o = 2 ,


x(t ) =
T

SIGNALS & SYSTEMS

10 t = 10t T
1

Cn =

1 x(t )e jnot dt = 10te jn 2t dt T 0 0

Now

te j 2nt 1 1 e j 2nt = 10 dt j 2n 0 j 2n 0 1 j 2 nt j 2 nt e e = 10 + 2 2 j 2n 4 n 0 5 = j n 5 x(t ) = C n e j 2nt = j e j 2nt ; n = n n =


n = tan 1 = 2

n0 n0

Q.121 State and prove the following properties of DTFT. (i) Time shifting, frequency shifting (ii) Conjugate symmetry (iii) Time reversal. Ans:
x ( n ) X (e j )

(6)

(i) Time shfting: Frequency shifting: (ii)

x ( n no ) e j

no

X (e j )

e jo n x(n) X (e j ( o ) ) x * (n) X * (e j ) X (e j ) = X * (e j ) , x (n) real

(iii)

Even [x( n)] Re X (e j ) Odd [x(n)] j Im X (e j ) x ( n ) X (e j )

Q.122 Consider a stable causal LTI system whose input x(n ) and output y (n ) are related through second order difference equation 1 3 y (n ) y (n 1) + y (n 2 ) = 2 x(n ) . 8 4

1 Determine the response for the given input x(n ) = u (n ) 4 Ans:


76

(10)

AE06/AC04/AT04
n

SIGNALS & SYSTEMS

3 1 1 y (n) y (n 1) + y (n 2) = 2 x(n) = 2 u (n) 4 8 4 Taking DTFT on both sides 3 1 2 Y (e j ) e j Y (e j ) + e 2 j Y (e j ) = 1 j 4 8 1 e 4 2 j Y (e ) = 3 j 1 2 j 1 j 1 e + e 1 e 8 4 4 2 = j 1 1 j 1 j 1 e 1 e 1 e 2 4 4 2 = 2 1 j 1 j 1 e 1 e 2 4 8 4 2 = j j 1 1 1 j 2 1 1 e e 1 e 2 4 4 Taking inverse DFT 1 1 1 y (n) = 8 u (n) 4 u (n) 2(n + 1) u (n) 2 4 4
n n n

Q.123

A continuous time signal is x(t ) = 8 cos 200t (i) Determine the minimum sampling rate. (ii) If fs = 400 Hz, what is discrete time signal obtained after sampling? (iii) If fs = 150 Hz, what is discrete time signal obtained after sampling?

(8)

Ans:Here = 200 f = 100 Hz (i) Minimum sampling rate = 2 f = 2 100 = 200 Hz f 100 1 (ii) = = f s 400 4 1 n x(n) = 8 cos 2fn = 8 cos 2 n = 8 cos 4 2
(iii) Here
f 100 2 = = f s 150 3

x(n) = 8 cos 2fn = 8 cos 2

2 4n 2 4n n = 8 cos = 8 cos 2 n = 8 cos 3 3 3 3


(8)

Q.124 State and prove Parsevals theorem for continuous time periodic signal.

77

AE06/AC04/AT04

SIGNALS & SYSTEMS

Ans: Parsevals theorem: The Parsevals theorem states that the energy in the time-domain representation of a signal is equal to the energy in the frequency domain representation normalized by 2. Proof: The energy in a continuous time non-periodic signal is

E = x(t ) dt.

Since x(t ) = x(t ) x * (t ) , from the Fourier series we get


1 X * ( j )e jt d . 2 1 E= x (t ) X * ( j )e jt d dt 2 x * (t ) =

Hence,
1 2

Now interchanging the order of the integrations, we get


E=

X * ( j )

1 x (t )e jt dt d = 2

X * ( j ) X ( j ) d

1 2 X ( j ) d . 2 Thus, the energy in the time-domain representation of the signal is equal to the energy in the frequency-domain representation normalized by 2. =

Q.125 Compute the magnitude and phase of the frequency response of the first order discrete time LTI system given by equation (10) y (n ) Ay (n 1) = Bx(n ) where A < 1 . Ans: y ( n) = Ay ( n 1) + Bx( n )

h(n) = BA n u (n)
H (e j ) =

h ( n) e
2

j n

Since
1 Ae jn =

1 Ae

n = jn

1 Ae e = (1 A cos ) + jA sin
2

B
j

(1 A cos ) + ( A sin ) = 1 + A sin Angle (1 Ae ) = tan 1 AA cos


jn 1

2 A cos

H ( e j ) = and

B 1 + A 2 A cos
2

1 angle H (e j ) = Angle B tan

A sin . 1 A cos

Q.126 Determine the Fourier transform of unit step x(t ) = u (t ). Ans:


Fourier transform of x(t) is Thus Fourier transform of u(t) is
X ( ) =

(6)

x(t )e jt dt

78

AE06/AC04/AT04
X ( ) =

SIGNALS & SYSTEMS

u (t )e jt dt =

e jt dt =

1 . j

Q.127 By using convolution theorem determine the inverse Laplace transform of the following functions. (8) 1 1 (i) (ii) s2 s2 a2 s 2 (s + 1) Ans: 1 (i) F (s) = 2 2 = F1 ( s ) F2 ( s ) s (s a 2 )

where Thus Now

F1 ( s ) =

1 s2

and and
t

F2 ( s) =

1 s a2
2

f1 (t ) = t

f 2 (t ) =

1 sinh (at ) a

1 F ( s ) = f 1 (t ) f 2 ( )d
1 = (t ) sinh (a )d a 0

0 t

(ii) where Thus Now


t t

1 [sinh at 1] a3 1 F (s) = 2 = F1 ( s) F2 ( s) s ( s + 1) 1 1 F1 ( s ) = 2 and F2 ( s ) = ( s + 1) s =

f 1 (t ) = t
t 1

and

f 2 (t ) = e t

F ( s ) = f1 (t ) f 2 ( )d
0

= (t )e d = te d e d = (te t + t ) + (te t + e t 1) = t + e t 1
0 0 0

Q.128 Check the stability & causality of a continuous LTI system described as (s 2) H (s ) = (s + 2)(s 3) Ans:
s2 1 4 1 . = + ( s + 2)( s 3) 5 s + 2 s 3 The system has poles at s = -2, s = 3. Thus, the response of the system will be

(8)

Given

H (s) =

h1 (t ) =

4 2t 1 e u (t ) + e3t u (t ), > 3 5 5 4 2t 1 h2 (t ) = e u (t ) e3t u (t ), - 2 < < 3 5 5

79

AE06/AC04/AT04
4 1 h3 (t ) = e 2t u (t ) e 3t u ( t ), < 2 5 5

SIGNALS & SYSTEMS

Note that the response h1(t) is unstable and causal, h2(t) is stable and non-causal, h3(t) is stable and non-causal. Thus, the system cannot be both stable and causal simultaneously.

Q.129

Find the z -Transform X ( z ) and sketch the pole-zero with the ROC for each of the following sequences. (8) 1 1 (i) x(n ) = u (n ) + u (n ) 2 3
n n

1 1 (ii) x(n ) = u (n ) + u ( n 1) 3 2 Ans:


1 1 (i) x(n) = u (n) + u (n) 2 3 z 1 , u ( n) 1 2 z 2 n z 1 , u (n) 1 3 z 3 Thus,
n n n

Z >

1 2

z >

1 3

5 2 z z z z 12 X (z) = + = , 1 1 1 1 z z z z 2 3 2 3 1 1 x(n) = u (n) + u ( n 1) 3 2


n n

Z >

1 2

(ii)

z 1 , u (n) 1 3 z 3
n

z >

1 3

z 1 1 , z < u (n 1) 1 2 2 z 2 z z 1 z X (z) = = , 1 1 1 1 6 z z z z 3 2 2 3

1 1 < z < 3 2

Q.130 Determine the inverse z transform of x(z ) =


convergence re (i) z > 1, (ii) z <

z
3z 4 z + 1
2

if the regions of

1 1 , (iii) < z < 1, 3 3

(8)

80

AE06/AC04/AT04 Ans:

SIGNALS & SYSTEMS


1 1 z X (z ) z F ( z) = = 2 = z 3z 4 z + 1 2 z 1 z 1 3

(i) ROC is z > 1


x ( n) =
n 1 1 n 1 n 1 (1) u (n) u (n) = 1 u (n) 2 3 2 3

(ii) ROC is z <

1 3
1 1 n x(n) = (1) + 2 3
n

u ( n 1)

(iii) ROC is

1 < z <1 3
n n 1 1 1 1 n ( ) 1 u ( n 1 ) u ( n ) = u ( n 1 ) + u ( n) 2 2 3 3

x(n) =

Q.131

Consider the probability density function f X ( x ) = ae , where X is a random variable whose allowable value range from x = to x = + . Find (i) Cumulative distribution function FX ( x ). (ii) Relationship between a and b. (iii) P[1 X 2] [assume b = 6] (8) Determine mean, mean square and Variance.

b x

Ans:
(i)
f x ( x ) = ae
b x

Thus,

f ( x ) = ae bx , < x < 0 f x ( x) = 1 bx f 2 ( x ) = ae , 0 < x < x a Fx1 ( x) = ae bx dx = e bx , x < 0 b x a Fx 2 ( x ) = ae bx dx = 1 ebx , x > 0 0 b a bx x<0 e , Fx ( x ) = b a 1 e bx , x > 0 b

(ii) Now

aebx dx +

ae bx dx = 1

a a + = 1 2a = b = 6 (given) a = 3 b b

(iii) Now

P[1 X 2] =

3e 6 x dx = 0.5e 6 e 6 1 .
x<0 x>0

3e 6 x f x ( x) = 6 x 3e

81

AE06/AC04/AT04
Mean = E ( x) =

SIGNALS & SYSTEMS

x3e 6 x dx +
0

x3e 6 x dx = 0

x 2 e 6 x dx = x 2 e 6 x dx 0 = Variance of X = 2 = E (x 2 ) [E (x )]2 = 3 0 18

Q.132 Find the power spectral density for the cosine signal x(t ) = 8 cos[2 (3) t + 3] and also compute power in the signal. (8) Ans: Autocorrelation function of x(t )
1 R( ) = Lt T x(t ) x (t + )dt T T/ 2
T /2

= Lt T
= Lt T

1 8 cos(6t + / 3)8 cos[6 (t + ) + / 3]dt T / 2


T /2

/2

32 [cos(12t + 6 + 2 / 3) + cos 6 ]dt T T/ 2 = 0 + 32 cos 6 = 32 cos 6 PSD = F [R( )] = F [32 cos 6 ] = 32 [ ( 2 ) + ( + 6 )] Power in the signal is R (0) = 32 cos 6 = 0 = 32 W

82

Anda mungkin juga menyukai